[go: up one dir, main page]

0% found this document useful (0 votes)
342 views86 pages

Medical Students' Neurology Quiz

1. The document contains a 35 question multiple choice medical exam covering various topics in internal medicine and neurology. 2. Questions test knowledge of symptoms and syndromes, including autonomic nervous system functions, cranial nerve damage presentations, neurological disorders like Friedreich's ataxia, and localization of neurological lesions. 3. Answer choices are designed to distinguish between options that are characteristic or pathognomonic for certain conditions versus those that are not specific findings.

Uploaded by

Ahmad Sobih
Copyright
© © All Rights Reserved
We take content rights seriously. If you suspect this is your content, claim it here.
Available Formats
Download as DOCX, PDF, TXT or read online on Scribd
0% found this document useful (0 votes)
342 views86 pages

Medical Students' Neurology Quiz

1. The document contains a 35 question multiple choice medical exam covering various topics in internal medicine and neurology. 2. Questions test knowledge of symptoms and syndromes, including autonomic nervous system functions, cranial nerve damage presentations, neurological disorders like Friedreich's ataxia, and localization of neurological lesions. 3. Answer choices are designed to distinguish between options that are characteristic or pathognomonic for certain conditions versus those that are not specific findings.

Uploaded by

Ahmad Sobih
Copyright
© © All Rights Reserved
We take content rights seriously. If you suspect this is your content, claim it here.
Available Formats
Download as DOCX, PDF, TXT or read online on Scribd
You are on page 1/ 86

Internal Medicine II Gr.16-30 Midterm 01.12.

2022
1. A patient has bradycardia (heart rhythm lowering), blood pressure lowering,
asthmatic breath, myosis. Name the attack that withholds these symptoms.
A. sympathy-adrenal
B. vagoinsular
C. mixed
D. panic

2. A young woman has amenorrhea, diabetes insipidus, trophic disturbances,


insomnia. Choose from the list the name of the preliminary syndrome –
A. Cushing
B. Vaso-vegetative
C. Addison
D. Hypothalamic

3. From enumerated, what is specific for Lambert-Eaton myasthenic syndrome?


A. damage of the acetylcholine receptors in postsynaptic terminal;
B. damage of the voltage gated calcium channels in presynaptic terminal;
C. damage of the synaptic cleft.
D. disorder of acetylcholine and enzyme choline acetyl transferase cooperation.

4. From enumerated, what isn't specific for Jackson's alternation syndrome?


A. palsy of lower mimic muscles of face;
B. deviation of the tongue on the side of pathological process;
C. atrophy of half of the tongue;
D. spastic hemiplegia.

5. Both trigeminal neuralgia and atypical facial pain involve pain that may be – PreTest
A. Unilateral
B. associated with anesthetic patches
C. lancinating
D. paroxysmal

6. From enumerated, which component is not a part of myotatic reflex?


A. Renshaw cells.
B. myoneural membranes;
C. Golgi tendon apparatus;
D. Golgi- Macconi's corpuscles;
E. motoneurons;

7. The neurological abnormalities due to Friedreich's ataxia are following, except –


A. degeneration of posterior columns
B. hypertrophic obstructive cardiomyopathy
C. degeneration of spinocerebellar tracts
D. degeneration of piramidal tracts
E. ataxia due to autosomal dominant inheritance
8. Vascular dementia is due to following, except
A. small vessel disease
B. neurodegeneration
C. "lacunar” infarcts
D. post stroke condition

9. From enumerated, which is the pathognomonic sign of capsular hemiplegic


syndrome?
A. Wernike-Mann's posture;
B. lose of sensation.
C. qvadriplegia;
D. central palsy of n. hypoglossus;

10. The main functions of the ANS are all the following, except:
A. trophotropic (homeostasis)
B. ergotropic (adaptive processes)
C. phagotropic (detection infectious agents)

 The іуаіп functions of the autonomic nervous system


a) trophotropic—regulation of the activity of the internal organs, maintenance of the
stability of the internal environment of the organism — homeostasis;
b) ergotropic — provision of adaptive processes — all forms of the psychic and physical
activity of an organism.

11. From enumerated, what isn't characteristic for damage of facial nerve after
chorda tympani leaves the facial canal?
A. hyperacusia;
B. loss of taste on anterior 2/3-rd of tongue.
C. dryness of eye;
D. prosoplegia;

12. From enumerated, what isn't characteristic for damage of facial nerve outside
the stylomastoid foramen?
A. tearing;
B. vestibular ataxia,
C. prosoplegia,
D. deafness
E. loss of taste on anterior 2/3-rd of tongue.

13. From enumerated, in which case the quadrant anopsia isn't observed?
A. partial damage of the radiation of Gratiolet.
B. damage of sulcus calcarina's cuneus;
C. damage of the lateral geniculate body.
D. damage of gyrus lingualis;

14. Examination of the patient revealed central monoparesis of the right hand and
motor aphasia. Choose from the list localization of pathological process –
A. posterior limb of the internale capsule
B. left frontal lobe
C. middle part of the precentral gyrus
D. Broca's center

15. From enumerated, which is response for voluntary movements of eyes?


A. calcarine cortex,
B. nuclei of pons Varolli:
C. area 8 by Brodmann;
D. occipital lobe.

16. Indicate main clinical features of tabes dorsalis –


A. bulbar disturbances, Horner syndrome
B. meningeal symptoms, papilloedema
C. paraplegia, fibrillary twitching
D. sensitive ataxia, the absence of the knee and Achilles reflexes

17. Clinical features of Cushing syndrome are following, except –


A. easy bruising
B. thin lower extremities
C. central obesity
D. abdominal striae
E. arterial hypotension

18. From enumerated, what is specific for Brown-Sequard syndrome?


A. loss of deep sensation on the opposite side of pathological process;
B. peripheral palsy on the side and below of pathological process;
C. central palsy below and on the side of pathological process;
D. syringomielic anesthesia of segmental type on the opposite side from the process.

From enumerated, what isn’t specific for Brown-Sequard syndrome?


A. lost of deep sensation on the side of pathological process;
B. flaccid palsy on the side of pathological process;
C. central palsy below and the side of pathological process;
D. syringomielitic anesthesia of segmental type on the opposite side from the process.

From enumerated, what isn’t specific for Brown-Sequard syndrome?


A. peripheral palsy on the level and on the side of pathological process;
B. central palsy below and on the side of pathological process;
C. spinal conductive tabetic anesthesia below and on the opposite side
from process;
D. syringomielitic anesthesia of condactive type on the opposite side from
the process.

19. From enumerated, what isn't characteristic for damage of oculomotor nerve?
A. enophthalmus;
B. ptosis;
C. diplopia;
D. deviation of the eye outwards the nose.

20. From enumerated, what is characteristic for damage of trochlear nerve?


A. palsy of m. rectus inferior;
B. palsy of m. obliquus inferior.
C. palsy of m. obliquus superior;

21. From enumerated, which pathology are responsible for isolative loss of deep
sensation?
A. amyotrophic lateral sclerosis;
B. poliomyelitis.
C. myelitis;
D. tabes dorsalis;

22. Diagnostic criterion of migraine include:


A. bilateral headache filling as like a band
B. absence of nausea and vomiting
C. pulsating unilateral headache

23. From enumerated, damage of which structure didn't produce palsy?


A. anterior pyramidal tract;
B. peripheral motoneuron.
C. lateral cortico-spinal tract,
D. central motoneuron;

24. The synonym (name) of the parasympathetic nervous system is:


A. Thoracolumbal
B. hypotalamc pituitary
C. cholinergic

25. The neurotransmitters of autonomic nervous system are all the following, except:
A. acetycholine
B. epinethrine
C. noradrenalin
D. serotonin

26. Where is the localization of epileptic lesion in seizure with motor aura?
A. in the postcentral gyrus
B. in the occipital lobe
C. in the temporal lobe
D. in the frontal lobe
 or In the precentral gyrus

27. Vernicke-Korsakoff syndrome is characterized by following, except:


A. memory loss
B. small vessels disease
C. deficiency of thiamine (Vitamin B1)
D. confusion about time, place and person

28. From enumerated, what is specific for Myasthenia gravis?


A. damage of the voltage gated calcium channels in presynaptic terminal,
B. damage of the acetylcholine receptors in postsynaptic terminal;
C. decreased level of acetylcholine;
D. disorder of acetylcholine and enzyme choline acetyl transferase cooperation

29. For Pierre Marie's disease isn't typical –


A. cerebellar ataxia
B. loss of coordination in the arm
C. tendency of fall when walking
D. deformation of foot as "pes cavus”

30. Diagnostic criteria of Addison disease are following, except


A. Plasma ACTH level 19 PG/DL
B. hyperpigmentation
C. anemia and eosonophilla
D. plasma cortisol >19 MG/DL
E. low blood glucose in plasma

31. Which of the following is typically the most difficult type of pain to treat? –
A. tension headache
B. psychogenic pain
C. intermittent somatic pain
D. continuous visceral pain

32. From enumerated, were "medial lemniscus" is situated?


A. in pons Varolli
B. in medulla oblongata
C. in lateral column of spinal cord
D. between thalamus and nucleus caudatus;

33. From enumerated, what pathology often accompanied Syringomyelia?


A. Arnold-Chiari malformation;
B. Brown-Sequard syndrome;
C. exramedullary neurofibroma.
D. arterio-venous malformation;

34. Diagnostic criteria of Friedreich's ataxia are the following, except –


A. dysarthria within 5 years after onset of ataxia
B. cognitive dysturbances
C. onset before the age of 25 years
D. lower limb areflexia
E. decrease of proprioceptive sense in lower limbes

 Diagnosis. In most cases FA can be clearly distinguished from other disorders by its highly
characteristic clinical phenotype. There are typical diagnostic criteria:
a) progressive ataxia with an onset before the age of 25 years,
b) lower limb areflexia,
c) decrease of proprioceptive or vibration sense in lower limbs,
d) dysarthria within 5 years after onset of ataxia.

35. From enumerated, in which case spinal conductive anesthesia is observed?


A. damage of posterior horn of spinal cord.
B. damage of lateral column of spinal cord;
C. damage of posterior root of spinal cord;

36. Myelin of CNS formed by –


A. oligodendroglial cells
B. microglial cells
C. astrocytes
D. ependimal cells

37. From enumerated, which part of body voluntary movement's program is


represented in medial third of anterior central convolution (precentral gyrus)?
A. lower limb;
B. upper limb;
C. articulation;
D. head and neck.

\\\\ From enumerated, which part of body voluntary movement’s program is represented
in medial third of anterior central convolution (fissure)?
\\\ lower limb;
\\\ articulation;
\\ upper limb;
\\\ head.

38. In the clinical triad of ipsilateral Horner syndrome, which symptom is not
correct:
A. Diplopia
B. A constricted pupil
C. Ptosis
D. Enophthalmos

39. From enumerated, which are the symptoms of Parkinson disease?


A. ropulsions, monotonous speech;
B. central paresis, hyperkinesis.
C. low muscular tone, ataxia
D. peripheral paresis, low muscular tone:

 Degeneration of nerves in the basal ganglia that leads to tremors,


weakness of muscles, masklike facies, and slowness of movement.

 Choose the symptoms of Parkinson disease


a) central paresis of the extremities, hyperkinesis
b) propulsions, monotonous speech
c) the upper muscular tone, peripheral paresis of the extremities
d) central paresis of the extremities, hyperkinesis
e) peripheral paresis of the extremities, hyperkinesis

40. From enumerated, which spinal nerves' anterior branches form the plexus brachialis?
A. C5-Th2.
B. C2-C5;
C. C3-C7;
D. C5-C8;

41. From enumerated, what is observed according to damage of inferior 2/3-d's of


the anterior central fissure?
A. monoplegia;
B. brachio-facial palsy
C. paraplegia;
D. hemiplegia

42. From enumerated, what isn't characteristic for pseudo-bulbar palsy syndrome?
A. dysartria;
B. Dysphagia
C. uncontolled laughing or crying;
D. dysphonia;
E. anatria.

43. From enumerated, what isn't specific for Myasthenia gravis?


A. disease begins with weakness of lower limbs muscles;
B. a drooping eyelids,
C. a myasthenic crisis;
D. damage of postsynaptic terminal's acetylcholine receptors.

44. Which of the following is not correct due to sweat glands?


A. neurotransmitter of postganglionic neurons is adrenergic
B. neurotransmitter of preganglionic neurons is acetylcholine
C. innervated by sympathetic nervous system

45. Transection of the spinal cord is characterised by, except –


A. dysorders of pelvic organs
B. all kinds of sensation disturbansies begin from level of transection
C. spastic lower paraplegia
D. spinal condactive type of paraanesthesia
 Transection of the spinal cord is characterized by disorder of all kinds of sensitivity of
the conductive type below the tumor, movement disorders, disorders of pelvic organs
and trophism. The patient is threatened by bedsores and urosepsis.

46. From enumerated, at which levels are placed the lateral horns of spinal cord?
A. at the level of lumbar and sacral segments.
B. at the level of C6-Th2 segments;
C. at the level of cervical segments;
D. at the level Th2-Th12 segments;

47. From enumerated, in which case the bulbar palsy syndrome is observed?
A. due to damage of nuclei lower (caudal) cranial nerves - in one side of
medulla oblongata;
B. due to bilateral impairment of cranial nerves IX, X, Xl and XII;
C. damage of both cortico-nuclear tracts.

48. From enumerated, which pathology isn't characteristic for damage of the
extrapyramidal system?
A. intention tremor;
B. torsion dystonia.
C. parkinsonian tremor;
D. hemibalism;
E. athetosis;

49. From enumerated, were loss of sensation is observed due to damage of


median nerve?
A. the dorsal surface of hand;
B. on the palm side - I, lI and partial IlI fingers;
C. terminal phalanges of fingers.
D. on the palm side - IV, V and partial IlI fingers;
50. From enumerated, what isn't specific for Myer-Hubler's alternation syndrome?
A. spastic hemiplegia;
B. Bell's sign:
C. deviation of the eye outwards the nose.
D. palsy of mimic muscles of face;

51. From enumerated, which type of loss of sensation didn't refer to peripheral type?
A. mononeural;
B. root type;
C. segmental type;
D. polineural.

52. Adrenergic component of the sympathetic nervous system is:


A. Cholinesterase
B. Norepinephrine
C. Acetylcholine
D. Serotonin

53. From enumerated, what isn't observed in case of Gullain-Barre syndrome?


A. weakness of the lower extremities;
B. paresthesia (numbness, tingling) in limbs;
C. oculomotor cranial nerve damage.
D. weakness of the upper extremities;

19.\\\\ From enumerated, what isn’t observed in case of Gullain-Barre syndrom?


\\\ paresthesia (numbness, tingling) in limbs;
\\\ increased white cell count (pleocytosis) in CSF;
\\\ weakness of the lower extremities;
\\\ facial cranial nerve often is affected.

 Paresthesia (numbness, tingling) in fingers and toes, pain in the extremities along the
peripheral nerves may also appear. Later weakness in the lower extremities appears
which than spreads to the upper extremities and progresses to paralysis.

54. What is the most appropriate segments level of sympathetic ANS? –


A. spinal cord and medulla
B. hypothalamus
C. lateral horns cells of C4 - Th12 segments
D. lateral horns cells of Th2 - L2 segments

55. The motor aphasia is characterized by:


A. speech dysartria
B. disturbance of expressive speech
C. disturbance of speach comprehension
D. mutism
E. dysphonia

56. What structures of NS are damaged in amyothrophic lateral sclerosis?


A. spinal cord anterior and posterior roots
B. pyramidal tracts and anterior horns
C. spinal cord anterior and posterior horns
D. sensory tracts and peripheral nerves

57. Alzheimer disease is characterized by following, except:


A. senile plaques
B. "lacunar" infarcts
C. neurofibrillary tangles
D. neurodegeneration

58. From enumerated, what isn't observed in case of Gullain-Barre syndrome?


A. weakness of the lower extremities;
B. paresthesia (numbness, tingling) in limbs;
C. oculomotor cranial nerve damage.
D. weakness of the upper extremities;

19.\\\\ From enumerated, what isn’t observed in case of Gullain-Barre syndrom?


\\\ paresthesia (numbness, tingling) in limbs;
\\\ increased white cell count (pleocytosis) in CSF;
\\\ weakness of the lower extremities;
\\\ facial cranial nerve often is affected.

 Paresthesia (numbness, tingling) in fingers and toes, pain in the extremities along the
peripheral nerves may also appear. Later weakness in the lower extremities appears
which than spreads to the upper extremities and progresses to paralysis.

59. The synonyms (name) of the sympathetic nervous system are all the following,
except:
A. Cholinergic
B. Toracolumbal
C. Adrenergic

60. Diagnostic criterion of migraine include:


A. bilateral headache filling as like a band
B. pulsating unilateral headache
C. preserved phisical activity
D. absence of nausea and vomiting

Psychiatry
61. A 20-year-old woman tells the physician that sometimes she becomes
frightened when her room is dark because her computer looks like a lion lurking
in the corner. This is an example of:
A. an illusion
B. a neologism
C. a hallucination
D. a delusion
BRS Behavioral Science  An illusion is a misperception of a real external stimulus (e.g.,
a computer looking like a lion lurking in the corner in a darkened room). A hallucination
is a false sensory perception, and a delusion is a false belief not shared by others. An
idea of reference is the false belief of being referred to by others, and a neologism is a
new word invented by a psychotic person.

62. Which gender has higher prevalence of ADHD?


A. Prevalence is equal in both genders
B. Male
C. Female

63. What is first-line pharmacologic treatment for ADHD?


A. SSRIs
B. Atypical antipsychotics
C. Lithium
D. Stimulants

64. Which of the following is a chronic mood disturbance that can cause depressive
symptoms, but does not disrupt normal functioning?
A. Cyclothymic Disorder.
B. Dysthymic Disorder. (Persistant depressive disorder)
C. Dissociative disorder.
D. Personality disorder.
Website  Dysthymic Disorder: A form of depression in which the sufferer has
experienced at least 2 years of depressed mood for more days than not.

65. A 30-year-old man comes to the psychiatrist because he is anxious about his
new job. He notes that he previously held a job shelving books in the back of a
library, but because of budget cuts he has been forced to interact with
customers. He states he doesn’t like being around people and prefers being by
himself. He appears emotionally cold and detached during the interview. Which
is the most appropriate personality disorder.
A. Antisocial
B. Narcissistic
C. Histrionic
D. Borderline
E. Schizoid
PreTest
66. The most common type of hallucination seen in schizophrenia is:
A. Olfactory
B. Visual
C. Auditory
D. Gustatory

67. Antipsychotic medications can cause amenorrhea by increasing plasma


concentration of which hormone?
A. Estrogen
B. Prolactin
C. Growth hormone
D. Triiodothyronine

68. Which mood disorder could involve just manic episode without depressive episode?
A. Bipolar disorder 2
B. Dysthymia
C. Bipolar disorder 1
D. Cyclothymia
E. Major depressive disorder

69. Which of the following is a negative symptom of schyzophrenia?


A. Alogia
B. Delusion
C. Hallucination
D. Disorganized speech

Negative Symptoms
Affective flattening Decreased expression of emotion, such as lack of expressive gestures.
Alogia Literally “lack of words,” including poverty of speech and of speech content in
response to a question.
Asociality Few friends, activities, interests; impaired intimacy, little sexual interest.
Positive Symptoms
Hallucinations Auditory, visual, tactile, or olfactory hallucinations; voices that are commenting.
Delusions Often described by content; persecutory, grandiose, paranoid, religious; ideas of
reference, thought broadcasting, thought insertion, thought withdrawal.
Bizarre behavior Aggressive/agitated, odd clothing or appearance, odd social behavior,
repetitivestereotyped behavior

70. A 9-year-old boy is referred to a psychiatrist because of poor school performance. He


has been tested for learning disabilities but none are present, and he has an IQ in the
high normal range. The teacher reports that it is hard to hold his attention and that he
appears hyperactive and fidgety at school, which disrupts the class. However, he does
not purposefully go out of his way to disobey the teacher. His parents have noticed no
difficulties at home, but his soccer coach has noticed attention problems during practice,
and his Sunday school teacher has trouble teaching him because of distractibility. Which
of the following is the most likely diagnosis for this patient?
A. ADHD, combined type
B. ADHD, predominantly hyperactive type
C. ADHD, predominantly inattentive type
D. Oppositional defiant disorder

Case File  Attention deficit disorder, combined type. The diagnostic criteria for ADHD require
that the symptoms be present in more than one setting, usually (but not limited to) home and
school (answer E). However, this child seems to have evidence of symptoms observed at
school, church, and soccer. The child has prominent distractibility and hyperactivity, thus has
more than only the hyperactivity type (answer B) or inattentive type (answer C).

71. In bipolar II disorder. Major depressive episodes alternate with periods of.
A. Anxiety
B. Hypomania
C. Mania
D. Derealization

 Bipolar II disorder is similar to bipolar I disorder except that mania is absent in bipolar
II disorder, and hypomania (a milder form of elevated mood than mania) is the essential
diagnostic finding.
Bipolar I disorder is defined by the occurrence of mania (or a mixed episode).
72. A 5-year-old boy is brought to the psychiatrist because he has difficulty paying attention
in school. He fidgets and squirms and will not stay seated in class. It is noted that at
home he talks excessively and has difficulty waiting for his turn. His language and motor
skills are appropriate for his age. Which of the following is the most likely diagnosis?
A. Oppositional defiant disorder (ODD)
B. Attention-deficit hyperactivity disorder (ADHD)
C. Pervasive developmental disorder
D. Separation anxiety disorder
E. Mild mental retardation
PreTest  Excessive motor activity, usually with intrusive and annoying qualities, poor
sustained attention, difficulties inhibiting impulsive behaviors in social situations and on
cognitive tasks, and difficulties with peers are the main characteristics of ADHD,
combined type. Symptoms must be present in two or more settings (in this case, home
and school) and must cause significant impairment.

73. A 24-year-old graduate student is shunned by his fellow graduate students because
he is so "weird." He discusses topics like whether or not crystals are real forms of
galactic communication, and wonders out loud about the wiring in the building
where he works as to whether it has special meaning in its architecture. The student
denies having any kind of hallucination and his thought process is logical and goal-
directed. Which of the following is the most likely diagnosis?
A. Schizophrenia
B. Major depressive disorder with psychotic features
C. Schizoid personality disorder
D. Schizotypal personality disorder
E. Schizoaffective disorder

74. A 35-year-old woman has lived in a state psychiatric hospital for the past 10 years. She
spends most of her day rocking, muttering softly to herself, or looking at her reflection in
a small mirror. She needs help with dressing and showering, and she often giggles and
laughs for no apparent reason. Which of the following is the most likely diagnosis?
A. Schizophrenia
B. Bipolar disorder. Manic phase
C. Schizophreniform disorder
D. Schizoaffective disorder
E. Schizophrenia
PreTest  The essential characteristics of the disorganized type of schizophrenia are
disorganized speech and behavior, flat or inappropriate affect, great functional impairment,
and inability to perform basic activities such as showering or preparing meals. Grimacing, along
with silly and odd behavior and mannerisms, is common. Hallucinations and delusions, if
present, are fragmented and not organized according to a coherent theme. This subtype is
associated with poor premorbid functions, early insidious onset, and a progressive course
without remissions. The patient has obviously had the disorder too long for the diagnosis to be
schizophreniform, and the absence of mood symptoms makes a diagnosis of bipolar disorder
or schizoaffective disorder unlikely. Patients with a delusional disorder do not generally have
such a marked impairment in affect or function.
75. A 27-year-old woman has been feeling blue for the past 2 weeks. She has little
energy and has trouble concentrating. She states that 6 weeks ago she had
been feeling very good, with lots of energy and no need for sleep. She says that
this pattern has been occurring for at least the past 3 years, though the
episodes have never been so severe that she couldn’t work. Which of the
following is the most likely diagnosis?
A. Borderline personality disorder
B. Seasonal affective disorder
C. Cyclothymic disorder
D. Major depression, recurrent
E. Bipolar disorder, depressed
PreTest  Cyclothymic disorder is characterized by recurrent periods of mild
depression alternating with periods of hypomania. This pattern must be present for at
least 2 years (1 year for children and adolescents) before the diagnosis can be made.
During these 2 years, the symptom-free intervals should not be longer than 2 months.
Cyclothymic disorder usually starts during adolescence or early adulthood and tends to
have a chronic course. The marked shifts in mood of cyclothymic disorder can be
confused with the affective instability of borderline personality disorder or may suggest
a substance abuse problem.

76. A 24-year-old woman is hospitalized after a suicide gesture during which she
superficially slashed both her wrists. At the team meeting 3 days later, the male
resident argues that the patient has been doing quite well, seems to be responding
to therapy, and should be allowed to leave on a pass. The nursing staff angrily argues
that the resident is showing favoritism to the patient, and because of her poor
compliance with the unit rules, she should not be allowed out. The resident insists
the nurses are being punitive. The defense mechanism being used by the patient in
this scenario is a feature of which of the following personality disorders?
A. Narcissistic
B. Histrionic
C. Borderline
D. Antisocial
E. Dependent
PreTest  Patients with borderline personalities see others (and themselves) as wholly
good or totally bad, a psychological defense called splitting. They alternatively idealize
or devalue important figures in their lives, depending on their perceptions of the others’
intentions, interest, and level of caring. These dynamics often elicit similar responses in
the environment, with the individuals being idealized having a considerably better
opinion of the patient than those who are being devalued.

77. Which neurotransmiter is most likely involved in pathogenesis of schizophrenia


and Parkinson's disease?
A. Noradrenaline
B. GABA
C. Dopamine
D. Serotonin
78. A 19-year-old man is brought to the physician by his parents after he called them
from college, terrified that the Mafia was after him. He reports that he has eaten
nothing for the past 6 weeks other than canned beans because “they are into
everything––I can’t be too careful.” He is convinced that the Mafia has put cameras
in his dormitory room and that they are watching his every move. He occasionally
hears the voices of two men talking about him when no one is around. His
roommate states that for the past 2 months the patient has been increasingly
withdrawn and suspicious. Which of the following is the most likely diagnosis?
A. Delusional disorder
B. Schizoaffective disorder
C. Schizophreniform disorder
D. Schizophrenia
E. Phencyclidine (PCP) intoxication
PreTest  Schizophreniform disorder and chronic schizophrenia differ only in the
duration of the symptoms and the fact that the impaired social or occupational
functioning associated with chronic schizophrenia is not required to diagnose
schizophreniform disorder. As with schizophrenia, schizophreniform disorder is
characterized by the presence of delusions, hallucinations, disorganized thoughts and
speech, and negative symptoms. The total duration of the illness, including prodromal
and residual phases, is at least 1 month and less than 6 months. Approximately one-
third of patients diagnosed with schizophreniform disorder experience a full recovery,
while the rest progress to schizophrenia and schizoaffective disorder.
 Depending on the predominance of particular symptoms, four subtypes of
schizophrenia are recognized: paranoid, disorganized, catatonic, and residual. The man
in the question presents with the classic symptoms of paranoid schizophrenia. This
subtype of schizophrenia is characterized by prominent hallucinations and delusional
ideations with a relative preservation of affect and cognitive functions. Delusions are
usually grandiose or persecutory or both, organized around a central coherent theme.
Hallucinations, usually auditory, are frequent and related to the delusional theme.
Anxiety, anger, argumentativeness, and aloofness are often present. Paranoid
schizophrenia tends to develop later in life and is associated with a better prognosis.

79. Which extramyramidal side-effect of antipsychotic medications is characerised


by restlessness?
A. Akathisia
B. Tardive Dyskinesia
C. Pseudoparkinsonism
D. Dystonia

OLD EXAMS
Quiz: Neurology- Midterm I- 28/05/18
1. From enumerated, which is not belonged to the pathology of reflexes respond?
A. hypomimia;
B. anisoreflexia
2. From enumerated, which structure is responsible for perception of thermal stimuli?
A. Meissner’s corpuscles;
B. Merkel’s meniscus;
C. Krause’s cones;
D. Ruffini’s corpuscles.

3. From enumerated, what isn’t specific for Brown-Sequard syndrome?


A. peripheral palsy on the side and below of pathological process;
B. central palsy below and on the side of pathological process
C. syringomielitic anesthesia of condactive type on the opposite side from the process.
D. lost of deep sensation on the side of pathological process;
E. flaccid palsy on the side of pathological process;

4. From enumerated, what is observed according to damage of inferior 2/3-d's of


the anterior central fissure?
A. paraplegia;
B. monoplegia;
C. hemiplegia;
D. brachio-facial palsy

5. From enumerated, at which levels are placed the lateral horns of spinal cord?
A. at the level of cervical segments;
B. at the level of C6-Th2 segments;
C. at the level Th2-Th12 segments;
D. at the level of lumbar and sacral segments.

6. From enumerated, what isn’t specific for Myasthenia gravis?


A. a abnormal fatigue of oculomotor muscles;
B. is a acute autoimmune neuromuscular dise…
C. is T-cell-dependent autoimmune disease;

7. From enumerated, what is common in conditioned and unconditioned reflexes?


A. conditioned stimulus;
B. unconditioned stimulus;
C. constant reflex arch.

8. From enumerated, which structure of brain does not belong to motor zone?
A. anterior central convolution;
B. anterior frontal gyrus
C. superior frontal gyrus;
9. From enumerated, the spinal nerves of which segments does not form plexus?
A. brachial;
B. cervical;

10. From enumerated, which peripheral nerve does not origin from plexus sacralis?
A. n. cutaneus femoris lateralis;
B. n. ishiadicus;

11. From enumerated, were first neuron of deep and superficial sensation is situated?
A. intervertebral ganglia;
B. anterior horns of spinal cord;
C. posterior horns of spinal cord;
D. thalamus.

12. From enumerated, damage of which structure didn’t produce palsy?


A. Lateral cortico-spinal tract;
B. Anterior pyramidal tract;
C. Peripheral motoneuron
D. Central motoneuron;

13. From enumerated, which data represents the minimal speed of neural impulse
transmission?
A. 0.2 m/sec;
B. 10 m/sec;
C. 1.5 m/sec;
D. 20 m/sec.

14. From enumerated, what isn’t specific for Syringomyelia?


A. bilateral (two-sided) sensitive disorders
B. loss of the superficial sensation;
C. syringomielic anesthesia just below the affe…

15. From enumerated, in which segments is placed the plantar response reflex center?
A. L2 – L4;
B. S2 – S4;
C. L5 – S1

16. From enumerated, which type of loss of sensation didn't refer to peripheral type?
A. mononeural;
B. root type;
C. segmental type;
D. polineural.

17. From enumerated, which component is not a part of myotatic reflex?


A. motoneurons;
B. myoneural membranes;
C. Golgi- Macconi’s corpuscles;
D. Golgi tendon apparatus;
E. Renshaw cells.
18. From enumerated, which spinal nerves anterior branches form the plexus cervicalis?
A. C1-C4;
B. C5-C8
C. T1-T4
D. L1-L4
19. From enumerated, from which the brain isn’t developed?
A. diencephalon;
B. prosencephalon;
C. mesencephalon;
D. rombencephalon;

20. From enumerated, which type didn’t belong to simple sensation?


A. temperature;
B. kinesthetic;
C. vibration;
D. discrimination

Quiz: Neurology-Midterm II- 4/06/18


1. From enumerated, were in primary optic center situated center of pupils reaction to
light?
A. pulvinar of thalamus;
B. lateral geniculate body;
C. colliculus superior of tectum.

2. From enumerated, which layer of the cerebellum cortex transmits efferent impulses?
A. molecular cells;
B. purkinje cells;
C. basket cells;
D. granular cells.

3. From enumerated, which anatomical feature isn’t characteristic for optic nerve?
A. axons of first neurons formed optic nerve
B. first neurons of optic nerve are disposed in the fovea centralis of macula;

From enumerated, which anatomical feature is characteristic for optic tract?


A. formed by axons of bipolar neurons;
B. formed by axons of 3-rd neurons;
C. formed by axons of 1-st neurons.

4. From enumerated, which type of testing isn’t performed to reveal the deafness?
A. Schwabach test;
B. Rinne test;
C. Scarpa's test;
D. Weber's test.

5. From enumerated, which fibers of optic nerves cross through the chiasm?
A. only nasal half of retina of both optic nerves;
B. only temporal half of retina of bouth optic nerves;
C. temporal half of retina of one optic nerve and nasal half of other.

6. From enumerated, which is response for voluntary movements of eyes?


A. nuclei of pons Varolli;
B. area 8 by Brodmann;
C. calcarine cortex;
D. occipital lobe.

7. From enumerated, damage of which structure is the reason of Stuart-Holm’s


phenomena manifestation?
A. vermis of cerebellum
B. basal ganglias;
C. hemispheas of cerebellum

8. From enumerated, what isn’t characteristic for damage of ventral


posterolateral nucleus of thalamus?
A. dysestesia;
B. paraanesthesia.

From enumerated, which anatomical feature isn’t characteristic for optic


pathways after primary optic center?
A. are axons of 4-th neurons;
B. begins from ventral posterolateral nucleus of thalamus;
C. begins from lateral geniculate body.
D. terminates in calcarine fissure.

9. From enumerated, which system directly isn’t responsible for coordination of movements?
A. vestibular;
B. cerebellum;
C. optic;
D. deep sensation

10. From enumerated, which anatomical feature isn’t characteristic for hemianopsia?
A. defects of homonym visual fields of both eyes;
B. defects of heteronym visual fields of both eyes;
C. defects of ¼-th of visual fields of both eyes.

11. From enumerated, what is observed in case of impulses full block in the optic nerve?
A. amblyopia;
B. cortical blindness;
C. scotoma;
D. amaurosis;
E. visual hallucinations.

12. From enumerated, damage of which structure causes the loss of pupil's
reacting to light and accommodation?
A. oculomotor nerve:
B. Perlia's nucleus;
C. abducens nerves;
D. trochlear nerves;
E. Edinger-Westphal nuclei.

13. From enumerated, what is observed an case of damage of optic cortex center?
A. Wamblyopia;
B. Whemianopsia;
C. cortical blindness ,
D. Mamaurosis;
E. visual hallucinations.

14. From enumerated, which structure didn't belong to auditory system?


A. Wexternal ear;
B. middle ear;
C. lateral ear;
D. Winner ear.

15. From enumerated, what is characteristic for damage of trochlear nerve?


A. palsy of m. obliquus superior;
B. palsy of m. rectus inferior;
C. palsy of m. obliquus inferior,

16. From enumerated, which symptoms are of pallidar system impairment?


A. bradykinesia, micrographia, a high muscular tone;
B. Central paresis, hyperkinesis.

17. From enumerated, which pathology isn’t characteristic for damage of the
extrapyramidal system?
A. Parkinsonian tremor
B. Intention tremor
C. Hemibalism
D. Torsion dystonia
E. Athetosis

18. From enumerated, which anatomical feature isn’t characteristic for 4-th
neurons of optic system?
A. pass through the anterior limb of the internal capsule;

19. From enumerated, what isn’t characteristics for damage of facial nerve outside
the stylomastoid foramen?
A. vestibular ataxia
B. Tearing
C. Prosoplegia
D. Loss of taste on anterior 2/3 of tongue
E. Deafness
20. From enumerated, which anatomical feature isn’t characteristic for facial nerve?
A. makes outer or external knee;
B. makes inner knee;
C. innervates all mimic muscles of face;
D. leaves skull by stylomastoid foramen.

Quiz: Neurology- Midterm III- 11/06/18


1. Appearance of blood in CSF is typical for –
A. Meningitis
B. Subarachnoid hemorrhage
C. Status epilepticus
D. Ishemic stroke

2. Gold standard for diagnosis of cerebrovascular malformation is –


A. CT of the brain
B. Cerebral angiography
C. MRI of the brain
D. Ultrasound dopplerography of the vessels

3. Urgent measures for comatose patient include


A. Correction of possible hypoglicemia
B. Neurovizualisation
C. Detailed neurological examination
D. Correction of hypotheermy

Urgent measures for comatose patient include –


A. Permeability of airway system (A), adequate breathing (B), stable circulation (C)
B. Neurovizualisation
C. Lumbalur puncture
D. Correction of hypotheermy

4. From list, which tumor belongs to subtentorial?


A. medullar tumor
B. parietal lobe's tumor
C. frontal lobe's tumor
D. IV ventricle's tumor
E. occipital lobe's tumor

5. From listed below causes, most frequently stroke is due to –


A. Cardiogenic emboli
B. Big artery atherothrombosis and thromboembolism
C. Small vessel disease
D. Arteriitis and arterial dissection

6. Vascular dementia is due to following, except


A. small vessel disease
B. neurodegeneration
C. "lacunar” infarcts
D. post stroke condition

7. The synonyms (name) of the autonomic nervous system are all the following,
except:
A. somatic
B. vegetatic
C. visceral

8. The neurotransmitters of autonomic nervous system are all the following, except:
A. acetycholine
B. epinethrine
C. noradrenalin
D. serotonin

9. What is the most appropriate segments level of sympathetic ANS? –


A. spinal cord and medulla
B. hypothalamus
C. lateral horns cells of C4 - Th12 segments
D. lateral horns cells of Th2 - L2 segments

10. The most striking neurologic complication of von Economos encephalitis


(encephalitis lethargica), a type of encephalitis that occurred in epidemic
proportions along with viral influenca between 1917 and 1928 was –
A. Blindness
B. Hearing loss
C. Paraplegia
D. Parkinsonism

11. Coma, caused by supratentorial damage can be due to –


A. Tumor of frontal pole
B. tumor of the temporal lobe
C. Brain stem hemorrhage
D. Subdural and epidural hematoma

12. A 62-year old man with a history of myocardial infarction awakens with a
dense right sided hemiplegia. His eyes are tonically deviated to the left and he
does not respond to threat on the right side of his visual field. He appears to be
alert and responds to pain on the left side of his body. His speech is
unintelligible and nonfluent and he follows no instructions. Efforts to get him to
repeat simple phrases consistently fail. Pick the language disturbance that best
explains the clinical picture
A. Broca’s aphasia
B. Wernicke’s aphasia
C. Anomic aphasia
D. Global aphasia

13. A 55-year-old woman has a progressive dementia over the past year. Over the
last 3 months she has developed dysarthria, myoclonus intention tremor and
hyperreflexia. Her CSF VDRL is positive. This patient’s symptoms are being
caused by which of the following? –
A. A response to penicillin treatment
B. An autoummune reaction
C. An acute meningoencephalitis
D. A chronic meningoencephalitis

14. Stroke of the middle cerebral artery cause –


A. Sensomotor l hemiparesis with prevalence of weakness in the leg
B. Contralateral hemiplegy, hemisensory deficit, homonymous hemianopsia
, aphasia or gnostic disrturbances
C. Syndrome of Wallenberg
D. Tetraplegy, damage of cranial nerves, altered conscious

15. The synonyms (name) of the sympathetic nervous system are all the following, except:
A. Cholinergic
B. Toracolumbal
C. Adrenergic

16. Clinical features of Cushing syndrome are following, except –


A. easy bruising
B. thin lower extremities
C. central obesity
D. abdominal striae
E. arterial hypotension

17. Which of the following is typically the most difficult type of pain to treat? –
A. Psychogenic pain
B. Intermittent somatic pain
C. Continuous visceral pain
D. Intermittent neuropathic pain

18. Among listed below conditions, subtentorial cause of coma state can be attributed to –
A. Anterior cerebral artery infarcion
B. Hypoglicemia
C. Lobar hemorrhage
D. Cerebellar infarction
19. For myelitis transversa it is typical presence of –
A. Paraplegia and anesthesia
B. Hemiplegia and hemianesthesia
C. Alternatrive syndroms
D. Dissociative sensory disturbances.

20. Senso-motor cerebral stroke involve –


A. Only hemisensory disturbances
B. Isolated hemiparesis without other signs
C. Coexistance of hemiparesis and hemisenory disturbances without other signs
D. Paresis of the leg and ipsilateral ataxia

Quiz: Neurology- Midterm IV- Version 2- 18/06/18


1. Average prevalence of epilepsy in the world is:
A. 1%
B. 3%
C. 0.5%
D. 5%

2. In patients with multiple sclerosis stable neurological deficit is due to –


A. axonal damage
B. eosinophilic pleocytosis
C. disturbance between T-kiliers and T helpers ratio
D. periventricular demyelinisation

3. Choose from the list, what belongs to clinical sign of amyotrophic lateral sclerosis?
A. urination disturbances
B. sensitive disturbances
C. fibrillary twitching in the impaired muscles
D. alternating syndromes

4. In acute transverse myelitis –


A. section of spinal cord revealed damage of all spinal tracts
B. signs of spinal cord damage are asymmetric
C. damage of spinal cord destroy only several tracts
D. demyelinating areas extend to the pons and cerebellum

5. What isn't familiar to myotonia congenital?


A. it is Thomsen's disease
B. a tonic spasm of muscle after forceful voluntary contraction
C. defects in the voltage-dependent chloride channel gene
D. involvement in process of smooth and cardiac muscles

6. From list, which isn't early symtom of MS?


A. dizziness and unsteadiness
B. bulbar disorders
C. retrobulbar neuritis
D. absence of abdominal reflexes
E. decreasing of viobration sense

7. What is observed according to irritation of the right postcentral gyrus superior part?
A. asteriognosia in right upper limb
B. loss of compound sensation in left upper limb
C. central palsy in right lower limb
D. sensory Jackson in right lower limb

8. Choose from the list what belongs to generalized seizures?


A. absences
B. adversive
C. psyhomotor
D. simple sensory

9. What diagnostic method can prove the diagnosis of epilepsy?


A. CT scanning
B. MRI
C. electroencephalography
D. radioisotope

10. The man with 15 year duration of epilepsy referred to urgent department with
generalized tonic-clonic seizures with continous epileptic seizures. The drug of
first choice is:
A. Midazolame
B. Diazepame
C. Alprazolame
D. Ethosuximide

11. The following is the sign of cryptogenic epilepsy:


A. Non genetic disorders confirmed by neuroimagi
B. The developmental disorder of brain
C. Genetic predisposition
D. Focal epileptic seizures due to brain disorder not confirmed by neuroimaging

12. New attack of multiple sclerosis can be defined if time between onset of two
attacks is not less than -
A. 10 days
B. 15 days
C. 20 days
D. 30 days

13. Choose from the list what didn't belong to partial (focal) seizures:
A. absence
B. adversive
C. motor Jackson's

14. Relapse of multiple sclerosis is identified as acute neurological defecit lasting


not less than –
A. 2 hours
B. 10 hours
C. 4 hours
D. 24 hours

15. Choose from the list what didn't characterize temporal epilepsy?
A. derealization feeling "déjà vu"
B. distorted perceptions of time
C. feelings of tingling or like pins and needles
D. impaired consciousness

Choose from the list what characterize temporal epilepsy?


A. tonic seizures
B. distorted perceptions of time
C. twitching in the limbs
D. loss of consciousness

16. The neurological abnormalities due to Friedreich's ataxia are following, except –
A. degeneration of spinocerebellar tracts
B. hypertrophic obstructive cardiomyopathy
C. degeneration of posterior columns
D. degeneration of piramidal tracts
E. type of ataxias with autosomal dominant inheritance

17. During a head trauma a patient lost consciousness. In 5 min he didn't remember
the events before and after the trauma. On examination there are anisoreflexia
of the knee reflexes, weakness of the convergence of the eye balls, decreasing of
the corneal reflexes. What is the preliminary diagnosis? –
A. brain concussion
B. brain contusion
C. brain compression
D. epidural hematoma

18. What structures of NS are damaged in amyothrophic lateral sclerosis?


A. spinal cord anterior and posterior horns
B. sensory tracts and peripheral nerves
C. pyramidal tracts and anterior horns
D. spinal cord anterior and posterior roots

19. The following drug of choose for the treatment of myoclonic seizures is –
A. valproic acid
B. ethosuximide
C. clonazepame

20. Choose from the list what characterize serial epileptic seizures?
A. adversive seizures
B. atonic seizures
C. postictal period
D. acidosis

Old Exams:
Quiz: Neurology- Midterm III- 11/06/18
No. Question Correct Answer
1 Appearance of blood in CSF is typical for - subarachnoid hemorrhage
Gold standard for diagnosis of cerebrovascular
2 MRI of the brain
malformation is -
3 Urgent measures for comatose patient include - correction of possible hypoglicemia
4 From list, which tumor belongs to subtentorial? medullar tumor
5 From listed below causes, most frequently stroke is due to - big arteries atherothrombosis and thrombo
6 Vascular dementia is due to following, except: neurodegeneration
The synonyms (name) of the autonomic nervous system are
7 somatic
all the following, except:
The neurotransmitters of autonomic nervous system are all
8 serotonin
the following, except:
What is the most appropriate segments level of sympathetic
9 lateral horns cells of Th2 – L2 segments
ANS? -
10 Coma, caused by supratentorial damage can be due to - Subdural and epidural hematoma
The most striking neurologic complication of Economo’s
encephalitis (encephalitis lethargic - a type of encephalitis
11 parkinsonism
that occurred in epidemic proportions along with viral
infection between 1917 and 1928) was -
A 62-year old man with a history of myocardial infarction
awakens with a dense right sided hemiplegia. His eyes are
tonically deviated to the left and he does not respond to
threat on the right side of his visual field. He appears to be
12 alert and resounds to pain on the left side of his body. His global aphasia
speech is unintelligible and non fluent and he follows no
instructions. Efforts to get him to repeat simple phrases
consistently fail. Pick the speech disturbance that best
explains the clinical picture -
A 55 year-old woman has a progressive dementia over the
past year. Over the last 3 months she has developed
13 dysarthria, myoclonus, intention tremor and hyperreflexia. a chronic meningoencephalitis
Her CSF VDRL(test for syphilis) is positive. This patient’s
symptoms are being caused by which of the following? -
14 Stroke of the middle cerebral artery cause - contralateral hemiplegy, hemisensory defi
The synonyms (name) of the sympathetic nervous system
15 cholinergic
are all the following, except:
16 Clinical features of Cushing syndrome are following, except – arterial hypotension
Which of the following is typically the most difficult type of
17 psychogenic pain
pain to treat? -
Among listed below conditions, subtentorial cause of coma
18 cerebellar infarction
state can be due to: anterior cerebral artery infarcion
19 For myelitis transverses it is typical presence of - paraplegia and paraanesthesia
20 Senso-motor cerebral stroke involve - coexistance of hemiparesis and hemisenory
Quiz: Neurology- Midterm IV- Version 2- 18/06/18
No. Question Correct Answer
1 Average prevalence of epilepsy in the world is: 1%
In patients with multiple sclerosis stable neurological deficit is
2 axonal damage
due to -
Choose from the list, what belongs to clinical sign of
3 fibrillary twitching in the impaired muscles
amyotrophic lateral sclerosis?
4 In acute transverse myelitis - section of spinal cord revealed damage of
al
5 What isn’t familiar to myotonia congenital? involvement in process of smooth and cardi
6 From list, which are not early symtoms of MS? bulbar disorders
What is observed according to irritation of the right
7 sensory Jackson in right lower limb
postcentral gyrus superior part?
8 Choose from the list what belongs to generalized seizures? absences
9 What diagnostic method can prove the diagnosis of epilepsy? electroencephalography
The man with 15 year duration of epilepsy referred to urgent
10 department with generalized tonic-clonic seizures with diazepame
continous epileptic seizures. The drug of first choice is:
11 The following is the sign of cryptogenic epilepsy: focal epileptic seizures due to brain
disorder
New attack of multiple sclerosis can be defined if time
12 30 days
between onset of two attacks is not less than -
Choose from the list what didn’t belong to partial (focal)
13 absence
seizures?
Relapse of MS is identified as acute neurological deficit
14 24 hours
lasting not less than -
Choose from the list what didn’t characterize temporal
15 feelings of tingling or like pins and needles
epilepsy?
The neurological abnormalities due to Friedreich’s ataxia are type of ataxias with autosomal dominant
16
following, except – inheritance
During a head trauma a patient lost consciousness. In 5 min
he didn’t remember the events before and after the trauma.
17 On examination there are anisoreflexia of the knee reflexes, brain concussion
weakness of the convergence of the eye balls, decreasing of
the corneal reflexes. What is the preliminary diagnosis? –
What structures of NS are damaged in amyothrophic lateral
18 pyramidal tracts and anterior horns
sclerosis?
The following drug of choose for the treatment of myoclonic
19 valproic acid
seizures is -
Choose from the list what characterize serial epileptic
20 postictal period
seizures?
Quiz: Neurology- Midterm III- 11/06/18
No. Question Correct Answer
1 Appearance of blood in CSF is typical for - subarachnoid hemorrhage
Gold standard for diagnosis of cerebrovascular
2 MRI of the brain
malformation is -
3 Urgent measures for comatose patient include - correction of possible hypoglicemia
4 From list, which tumor belongs to subtentorial? medullar tumor
5 From listed below causes, most frequently stroke is due to - big arteries atherothrombosis and thrombo
6 Vascular dementia is due to following, except: neurodegeneration
The synonyms (name) of the autonomic nervous system are
7 somatic
all the following, except:
The neurotransmitters of autonomic nervous system are all
8 serotonin
the following, except:
What is the most appropriate segments level of sympathetic
9 lateral horns cells of Th2 – L2 segments
ANS? -
10 Coma, caused by supratentorial damage can be due to - Subdural and epidural hematoma
The most striking neurologic complication of Economo’s
encephalitis (encephalitis lethargic - a type of encephalitis
11 parkinsonism
that occurred in epidemic proportions along with viral
infection between 1917 and 1928) was -
A 62-year old man with a history of myocardial infarction
awakens with a dense right sided hemiplegia. His eyes are
tonically deviated to the left and he does not respond to
threat on the right side of his visual field. He appears to be
12 alert and resounds to pain on the left side of his body. His global aphasia
speech is unintelligible and non fluent and he follows no
instructions. Efforts to get him to repeat simple phrases
consistently fail. Pick the speech disturbance that best
explains the clinical picture -
A 55 year-old woman has a progressive dementia over the
past year. Over the last 3 months she has developed
13 dysarthria, myoclonus, intention tremor and hyperreflexia. a chronic meningoencephalitis
Her CSF VDRL(test for syphilis) is positive. This patient’s
symptoms are being caused by which of the following? -
14 Stroke of the middle cerebral artery cause - contralateral hemiplegy, hemisensory defi
The synonyms (name) of the sympathetic nervous system
15 cholinergic
are all the following, except:
16 Clinical features of Cushing syndrome are following, except – arterial hypotension
Which of the following is typically the most difficult type of
17 psychogenic pain
pain to treat? -
Among listed below conditions, subtentorial cause of coma
18 cerebellar infarction
state can be due to: anterior cerebral artery infarcion
19 For myelitis transverses it is typical presence of - paraplegia and paraanesthesia
20 Senso-motor cerebral stroke involve - coexistance of hemiparesis and hemisenory
Quiz: Neurology- Midterm I- 28/05/18
No. Question Correct Answer
From enumerated, which is not belonged to the pathology of
1 hypomimia;
reflexes respond?
From enumerated, which structure is responsible for
2 Ruffini’s corpuscles
perception of thermal stimuli?
From enumerated, what isn’t specific for Brown-Sequard peripheral palsy on the side and below of
3
syndrome? pathological process;
From enumerated, what is observed according to damage of
4 brachio-facial palsy
inferior 2/3-d’s of the anterior central fissure?
From enumerated, at which levels are placed the lateral
5 at the level of C6-Th2 segments;
horns of spinal cord?
6 From enumerated, what isn’t specific for Myasthenia gravis? is a acute autoimmune neuromuscular
disea
From enumerated, what is common in conditioned and
7 unconditioned stimulus;
unconditioned reflexes?
From enumerated, which structure of brain does not belong
8 anterior frontal gyrus
to motor zone?
From enumerated, the spinal nerves of which segments does
9 brachial;
not form plexus?
From enumerated, which peripheral nerve does not origin
10 n. cutaneus femoris lateralis;
from plexus sacralis?
From enumerated, were first neuron of deep and superficial
11 intervertebral ganglia;
sensation is situated?
From enumerated, damage of which structure didn’t produce
12 anterior pyramidal tract;
palsy?
From enumerated, which data represents the minimal speed
13 0.2 m/sec;
of neural impulse transmission?
14 From enumerated, what isn’t specific for Syringomyelia? syringomielic anesthesia just below the affe
From enumerated, in which segments is placed the plantar
15 L5 – S1
response reflex center?
From enumerated, which type of loss of sensation didn’t
16 segmental type;
refer to peripheral type?
From enumerated, which component is not a part of myotatic
17 Golgi- Macconi’s corpuscles;
reflex?
From enumerated, which spinal nerves anterior branches
18 C1-C4;
form the plexus cervicalis?
19 From enumerated, from which the brain isn’t developed? diencephalon;
From enumerated, which type didn’t belong to simple
20 discrimination
sensation?
Quiz: Neurology-Midterm II- 4/06/18
No. Question Correct Answer
From enumerated, were in primary optic center situated
1 colliculus superior of tectum
center of pupils reaction to light?
From enumerated, which layer of the cerebellum cortex
2 purkinje cells;
transmits efferent impulses?
From enumerated, which anatomical feature isn’t
3 axons of first neurons formed optic nerve
characteristic for optic nerve?
From enumerated, which type of testing isn’t performed to
4 Scarpa’s test;
reveal the deafness?
From enumerated, which fibers of optic nerves cross through only nasal half of retina of both optic
5
the chiasm? nerves;
From enumerated, which is response for voluntary
6 area 8 by Brodmann;
movements of eyes?
From enumerated, damage of which structure is the reason
7 hemispheas of cerebellum
of Stuart-Holm’s phenomena manifestation?
From enumerated, what isn’t characteristic for damage of
8 paraanesthesia.
ventral posterolateral nucleus of thalamus?
From enumerated, which system directly isn’t responsible for
9 optic;
coordination of movements?
From enumerated, which anatomical feature isn’t
10 defects of ¼-th of visual fields of both eyes
characteristic for hemianopsia?
From enumerated, what is observed in case of impulses full
11 amaurosis;
block in the optic nerve?
From enumerated, damage of which structure causes the loss
12 oculomotor nerve;
of pupil’s reacting to light and accommodation?
From enumerated, what is observed in case of irritation of
13 visual hallucinations
optic cortex center?
From enumerated, which structure didn’t belong to auditory
14 lateral ear;
system?
From enumerated, what is characteristic for damage of
15 palsy of m. obliquus superior;
trochlear nerve?
From enumerated, which symptoms are of pallidar system bradykinesia, micrographia, a high
16
impairment? muscular tone;
From enumerated, which anatomical feature isn’t pass through the anterior limb of the
17
characteristic for 4-th neurons of optic system? internal capsule;
From enumerated, which pathology isn’t characteristic for
18 intention tremor;
damage of the extrapyramidal system?
From enumerated, what isn’t characteristic for damage of
19 vestibular ataxia;
facial nerve outside the stylomastoid foramen?
From enumerated, which anatomical feature isn’t
20 innervates all mimic muscles of face;
characteristic for facial nerve?
Quiz: Neurology- Final- 9/02/18
No. Question Correct Answer
1 The term agnosia refers to the (a) ? loss of the power to recognize sensory stim
2 This sign/reflex is used to diagnose meningitis: Brudzinski
3 Second order neuron in superficial sensitivity tract is In the dorsal horn of the spinal cord
localized:
4 Tabes dorsalis is a term that means? disease of CNS, usually caused by syphilis
The presence of dysdiadochokinesis suggests damage to the
5 cerebellar
following:
6 The term anarthria refers to the? inability to speak
Findings in a patient with serotonin syndrome include each of
7 miosis
the following EXCEPT
Brown-Sequard syndrome develops with the following
8 half transversal
damage of the spinal cord:
The ability to walk along a straight line is most often
9 cerebellar dysfunction
impaired with:
Which of the following terms is associated with degeneration
of nerves in the basal ganglia that leads to tremors,
10 Parkinson disease
weakness of muscles, masklike facies, and slowness of
movement?
11 Knee jerk reflex arc is at level of: L2-L4
12 Babinsky response usually develops with damage in: upper motor neuron
13 Disorders associated with myasthenia gravis include all of the above
14 Duchenne Muscular dystrophy is present at birth
Hemiplegia, hemianesthesia & hemianopia develop together
15 internal capsule
with disease in the:
16 Plastic hypertonia is characterized by phenomenon of: Cogwheel
Failure of muscle coordination, including unsteady
17 movements and staggering walk, due to disorders in the ataxia
cerebellum is called?
A patient with a head injury who has eye opening to speech,
18 no verbal responses and withdrawal motor responses would 8
have a Glasgow Coma Scale score of
If in a patient upper border of pain and temperature
sensitivity defect is determined at the level of dermatome
19 T6 or T7
T8, hence the level of spinal cord lesions localized in the
segment:
The respiratory pattern of a patient sustaining an acute C5
20 a rapid respiratory rate
spinal cord injury is characterised by
21 Second order neuron in deep sensitivity tract is localized: In the medulla oblongata
22 Myopathy clinical findings include all except depressed reflexes
The upper motor neuron impairment produces the following
23 spasticity
change of muscles tone:
Which part of the brain regulates heartbeat, breathing, and
24 brainstem
other vital functions?
A multi-trauma patient opens his eyes and withdraws to
painful stimuli, but does not respond to voice. He is moaning
25 8
but makes no comprehensible sounds. His Glasgow Coma
Score is
26 The term parethesia refers to? the sensation of numbness, pricking, burnin
27 Serotonin and dopamine are? neurotransmitters
A chronic disease characterized by a loss of the myelin
28 sheath, causing paresthesias, muscle weakness, and multiple sclerosis
unsteady gait is called?
Clinical features of autonomic neuropathy include all of the
29 pronounced sinus arrhythmia
following EXCEPT
An INCORRECT statement regarding the autonomic nervous heart rate responses are primarily mediated
30
system is that through the sympathetic nervous system
31 Demyelination is defined as? loss of protective sheath surrounding neuro
A patient presents with headache, morning vomiting and
double vision for three weeks. On examination nystagmus is
32 posterior fossa tumour
present when the eyes are turned to either side. The most
likely diagnosis is
33 The brain and the spinal cord make up the? central nervous system
34 GAG reflex arc is at nuclear level of ____ cranial nerve: IX и X CN
Which of the following is caused by a thrombus which
35 cerebrovascular accident
occludes an artery leading to or within the brain?
The inability to use or understand spoken or written language
36 aphasia
because of a brain lesion is known as?
The site of a lesion producing blindness with preservation of
37 Occipital lobe
the pupillary reflexes is:
38 General meningeal syndrome is characterized by All of the above
39 A patient with bitemporal hemianopia has a lesion in the: Optic chiasma
40 Spastic hypertonia is characterized by phenomenon of: Clasp knife

Old Q-Bank
\\\\ From enumerated, which proportions comprise the cervical, thoracic, lumbar, sacral
and coccygeal segments in spinal cord?
\\\ 8-10-7-5-2;
\\ 8-12-5-5-2;
\\\ 10-12-5-3-2;
\\\ 7-12-6-5-2.

\\\\ From enumerated, which tract does not belong to the descending pathways of the
spinal cord?
\\ Govers’;
\\\ lateral pyramidal;
\\\ Monakov’s;
\\\ Turk’s;
\\\ vestibulospinal.

\\\\ From enumerated, which spinal nerves’ anterior branches form the intercostal
nerves?
\\\ C5-Th1;
\\\ Th 2- Th 6;
\\ Th 3- Th 12;
\\\ Th 6-L2.

\\\ From enumerated, which peripheral nerve does not origin from plexus lumbalis?
\\\ n. genitofemoralis;
\\\ n. femoralis;
\\ n. cutaneus femoris posterior;
\\\ n. obturatorius.
\\\\ From enumerated, which system provides the muscular tone regulation on the level
of spinal cord?
\\ gama-loop;
\\\ voluntary motor analysator;
\\\ cerebellum;
\\\ extrapyramidal system.

\\\\ From enumerated, which structure is responsible for perception of pain?


\\\ corpuscles of Golgi-Mazzoni;
\\\ corpuscles of Vater-Pacini;
\\ free nerve endings;
\\\ hair cuff.

\\\\ From enumerated, which part of CNS have segmental building?


\\\ midbrain;
\\\ cerebellum;
\\ brainstem;
\\\ basal ganglia.

\\\\ From enumerated, what isn’t specific for Brown-Sequard syndrome?


\\\ lost of deep sensation on the side of pathological process;
\\\ flaccid palsy on the side of pathological process;
\\\ central palsy below and the side of pathological process;
\\ syringomielitic anesthesia of segmental type on the opposite side from the process.

\\\\ From enumerated, which structure didn’t pass axons of second neurons of simple
sensation?
\\\ lateral column of spinal cord;
\\\ medulla oblongata;
\\ posterior column of spinal cord;
\\\ anterior limb of internal capsule.

\\\\ From enumerated, from which the brain isn’t developed?


\\ diencephalon;
\\\ prosencephalon;
\\\ mesencephalon;
\\\\ rombencephalon;

\\\\ From enumerated, at which levels are placed the lateral horns of spinal cord?
\\\ at the level of cervical segments;
\\ at the level of C6-Th2 segments;
\\\ at the level Th2-Th12 segments;
\\\ at the level of lumbar and sacral segments.

From enumerated, which data represents the minimal speed of neural impulse
transmission?
\\ 0.2 m/sec;
\\\ 10 m/sec;
\\\ 1.5 m/sec;
\\\ 20 m/sec.
8.\\\\ From enumerated, what is common in conditioned and unconditioned
reflexes?
\\\ conditioned stimulus;
\\ unconditioned stimulus;
\\\ constant reflex arch.

\\\\ From enumerated, which component is not a part of myotatic reflex?


\\\ motoneurons;
\\\ myoneural membranes;
\\ Golgi- Macconi’s corpuscles;
\\\ Golgi tendon apparatus;
\\\ Renshaw cells.

\\\\ From enumerated, which structure is responsible for perception of thermal stimuli?
\\\ Meissner’s corpuscles;
\\\ Merkel’s meniscus;
\\\ Krause’s cones;
\\ Ruffini’s corpuscles.

\\\\ From enumerated, which type didn’t belong to simple sensation?


\\\ temperature;
\\\ kinesthetic;
\\\ vibration;
\\ diskrimination.

\\\\ From enumerated, were first neuron of deep and superficial sensation is situated?
\\ intervertebral ganglia;
\\\ anterior horns of spinal cord;
\\\ posterior horns of spinal cord;
\\\ thalamus.

From enumerated, according to function, which type of neural cells does not exist?
\\\ unipolar;
\\\ bipolar;
\\\ vegetatic;
\\ commissural

\\\\ From enumerated, which structure does not contain the neural cell?
\\\ pericarion;
\\\ axon;
\\\ dendrite;
\\ pigment;
\\\ tigerish matter.

\\\\ From enumerated, how are placed the ascending and descending tracts within the
spinal cord?
\\\ in one column;
\\\ in two paired columns;
\\ in three paired columns;
\\\ in three columns.
.
\\\\ From enumerated, which spinal nerve’s anterior branches form the lumbar plexus?
\\ L1-L4;
\\\ L1-S1;
\\\ Th 10-L1;
\\\ Th 12-L2.

\\\\ From enumerated, which neurons do not participate in formation of anterior horn?
\\\ alpha-small motoneurons;
\\\ alpha-large motoneurons;
\\\ Renshow cells;
\\ Clark-Shtilling cells;
\\\ gamma-motoneurons.

\\\\ From enumerated, which part of body voluntary movement’s program is represented
in medial third of anterior central convolution (fissure)?
\\\ lower limb;
\\\ articulation;
\\ upper limb;
\\\ head.

\\\\ From enumerated, which tract passes “decussacio pyramidarum” on the border of
medulla oblongata and spinal cord?
\\ lateral cortico-spinal;
\\\ reticullo-spinal;
\\\ anterior cortico-spinal.

\\\\ From enumerated, which type didn’t belong to compound sensation?


\\\ stereognostic;
\\\ two point spacing;
\\ vibration;
\\\ two point discrimination.

\\\\ From enumerated, were 3-rd neuron of deep and superficial sensation is situated?
\\\ intervertebral ganglia;
\\\ anterior horns of spinal cord;
\\\ posterior horns of spinal cord;
\\ thalamus;
\\\ medulla oblongata.

19.\\\\ From enumerated, what isn’t observed in case of Gullain-Barre syndrom?


\\\ paresthesia (numbness, tingling) in limbs;
\\\ increased white cell count (pleocytosis) in CSF;
\\\ weakness of the lower extremities;
\\\ facial cranial nerve often is affected.

\\\\ From enumerated, what is observed in case of tabetic anesthesia?


\\\ astereognosia;
\\\ loss of pain sensation;
\\\ loss of two point space sensation;
\\ incoordination of movements.

\\\\ From enumerated, which structure is not belonged to brainstem?


\\\ brain peduncules;
\\\ pons varoli;
\\ internal capsule;
\\\ medulla oblongata

\\\\ From enumerated, which tract does not belong to the ascending pathways of the
spinal cord?
\\\ Goll’s;
\\ Monakow’s;
\\\ Burdach’s;
\\\ spinothalamic;
\\\ Flexing’s.

\\\\ From enumerated, which peripheral nerve does not origin from plexus cervicalis?
\\\ n. occipitalis minor;
\\\ n. transversus colli;
\\ n. axillaris;
\\\ n. phrenicus.

\\\\ Ffrom enumerated, how many neurons comprise the corticomuscular pathway?
\\\ one;
\\ two;
\\\ three;
\\\ four.

From enumerated, which structure does not participate in formation of internal capsule?
\\\ nucleus caudatus;
\\\ striatum;
\\ nucleus dentatus.
\\\ thalamus.

\\\\ From enumerated, what is observed due to damage of superior 1/3-d of the
posterior central fissure?
\\\ loss of sensation in upper limb;
\\\ loss of sensation on head and neck;
\\ loss of sensation in lower limb.

\\\\ From enumerated, damage of which structure of spinal cord revealed as conductive
type of spinal anesthesia?
\\\ posterior root;
\\\ lateral horn;
\\\ posterior horn;
\\ posterior column;
\\\ anterior commissure.

\\\\ From enumerated, in which case global anesthesia is observed?


\\\ damage of posterior column of spinal cord;
\\ damage of posterior root of spinal cord;
\\\ damage of posterior horn of spinal cord;
\\\ damage of lateral column of spinal cord.

\\\\ From enumerated, were second neuron of deep sensation is situated?


\\\ posterior horns of spinal cord;
\\ medulla oblongata.
\\\ anterior roots of spinal cord;
\\\ thalamus.

\\\\ From enumerated, how many segments are in spinal cord?


\\\ 26-28 segments;
\\\ 28-29 segments;
\\\ 30-31 segments;
\\ 31-32 segments.

\\\\ From enumerated, what isn’t specific for Brown-Sequard syndrome?


\\\ peripheral palsy on the level and on the side of pathological process;
\\\ central palsy below and on the side of pathological process;
\\ spinal conductive tabetic anesthesia below and on the opposite side from process;
\\\ syringomielitic anesthesia of condactive type on the opposite side from the process.

\\\\ From enumerated, which data represents the maximal speed of neural impulse
transmission?
\\\ 200 m/sec;
\\ 120 m/sec;
\\\ 70 m/sec;
\\\ 40 m/sec.

\\\\ From enumerated, which peripheral nerve does not origin from plexus brachialis?
\\\ n. medianus;
\\\ n. radialis;
\\ n. intercostalis;
\\\ n. ulnaris.

\\\\ From enumerated, which symptom is expressed in upper motor neuron damage?
\\\ muscular atrophy;
\\\ fasciculation;
\\\ sensorial damage;
\\\ tremor;
\\ spastic monoparesis.

\\\\ From enumerated, which tract does not pass the posterior limb of internal capsule?
\\\ pyramidal;
\\ corticonuclear;
\\\ axons of the 4th neuron of optical analysator;
\\\ bulbothalamic;
\\\ spinothalamic.

\\\\ From enumerated, which tracts decussate in medulla oblongata?


\\ tracts of deep sensation;
\\\ tracts of superficial sensation;
\\\ Fleqsing’s tracts;
\\\ thalamo-cortical tracts.

\\\\ From enumerated, were second neuron of superficial sensation is situated?


\\ posterior horns of spinal cord;
\\\ medulla oblongata.
\\\ pons Varolli;
\\\ thalamus optici.

\\\\ From enumerated, which type of loss of sensation refer to spinal type?
\\\ root type;
\\\ mononeural;
\\ conductive.

\\\\ From enumerated, what is observed in case of tabetic anesthesia?


\\\ astereognosis;
\\\ increse of muscle tone of extrapiramidal type;
\\ incoordination of movements;
\\\ hyperreflexy.

\\\\ From enumerated, were loss of sensation is observed due to damage of median
nerve?
\\\ the dorsal surface of hand;
\\ on the palm side – I, II and partial III fingers;
\\\ on the palm side – IV, V and partial III fingers;
\\\ terminal phalanges of fingers.

\\\\ From enumerated, which tract’s damage, in spinal cord, is a reason of sensitive
ataxia’s manifestation?
\\\ pyramidal;
\\\ Gowers’;
\\ bulbothalamic;
\\\ rubrospinal.

\\\\ From enumerated, which tract leave cerebellar by superior cerebellar peduncle?
\\ cerebellorubralis;
\\\ reticulospinalis;
\\\ spinocerebellaris ventralis.

\\\\ From enumerated, with which structure the cerebellum peduncles didn’t contact?
\\\ midbrain;
\\\ pons Varolli;
\\\ nuclei of pons;
\\ inferior colliculus;
\\\ medulla oblongata.

\\\\ From enumerated, in which case the bulbar palsy syndrom is observed?
\\ due to bilateral impaurment of cranial nerves IX, X, XI and XII;
\\\ due to damage of nuclei lower (caudal) cranial nerves – in one side of medulla
oblongata;
\\\ damage of both cortico-nuclear tracts.

\\\\ From enumerated, what isn’t characteristic for pseudo-bulbar palsy syndrom?
\\\ dysphagia;
\\\ uncontollable laughing or crying;
\\\ dysphonia;
\\\ dysartria;
\\ anatria.

\\\\ From enumerated, which anatomical structure didn’t belong to the labyrinth?
\\\ utriculus;
\\\ sacculus;
\\\ semicircular canals;
\\ spiral ganglion.

\\\\ From enumerated, which range of acoustic waves can be percept by human’s ear?
\\\ 10 – 25 000 Hz;
\\\ 10 – 30 000 Hz;
\\\ 20 – 35 000 Hz;
\\ 20 – 40 000 Hz.

\\\\ From enumerated, what isn’t observed in case of peripheral paralysis of facial
nerve?
\\\ Bell’s sign;
\\\ lagophtalmus;
\\ atrophy of half of tongue;
\\\ asymmetry of face;
\\\ “crocodile tears”.

\\\\ From enumerated, which is response for jointly (conjugate) movements of eyes?
\\\ oculomotor nerves;
\\\ abducens nerves;
\\\ trochlear nerves;
\\ medial longitudinal fasciculus.

\\\\ From enumerated, in which case the damage of cranial nerve is responsible for
hyperacusis on the side of the pathologic process?
\\\ V cr. n.;
\\ VII cr. n.;
\\\ VIII cr. n.
\\\ IX cr. n.;
\\\ X cr. n.;
\\\ XII cr. n.

\\\\ From enumerated, which tract’s damage is a reason of cerebral ataxia’s


manifestation?
\\\ bulbothalamic;
\\\ pyramidal;
\\ fronto-ponto-cerebellaris;
\\\ rubrospinal;
\\\ Flexing’s.

\\\\ From enumerated, which tract’s damage is a reason of cerebellar ataxy’s


manifestation?
\\ Flexsing’s;
\\\ spinothalamic;
\\\ bulbothalamic;
\\\ tectospinal.

\\\\ From enumerated, from which structure cerebellar didn’t get afferent impulsation?
\\\ nuclei graculis and cuneatus;
\\\ vestibular (Deiters) nucleus;
\\ pyramids of medulla oblongata;
\\\ inferior olives.

\\\\ From enumerated, what isn’t characteristic for bulbar palsy syndrom?
\\\ dysphagia;
\\ dystonia;
\\\ dysphonia;
\\\ dysartria;
\\\ anatria.

\\\\ From enumerated, which muscle isn’t innervated by XI cranial nerve?


\\\ sternocleidomastoid;
\\ deltoid;
\\\ trapezius.

\\\\ From enumerated, which muscle isn’t innervated by motor branches of the trigeminal
nerve?
\\\ masseter;
\\\ temporalis;
\\ buccalis;
\\\ medial pterygoid;
\\\ lateral pterygoid.

\\\\ From enumerated, what isn’t specific for Myer-Hubler’s alternation syndrome?
\\\ Bell’s sign;
\\\ palsy of mimic muscles of face;
\\\ spastic hemiplegia;
\\ deviation of the eye outwards the nose.

16. \\\\ From enumerated, in which case the damage of cranial nerve is responsible for
deviation of the eye towards the nose?
\\\ III cr. n.;
\\\ IV cr. n.;
\\\ V cr. n.;
\\ VI cr. n.;
\\\ VII cr. n.

\\\\ From enumerated, in which case the quadrant anopsia isn’t observed?
\\ damage of the lateral geniculate body;
\\\ damage of gyrus lingualis;
\\\ damage of sulcus calcarina’s cuneus;
\\\ partial damage of the radiation of Gratiolet.

\\\\ From enumerated, which anatomical feature isn’t characteristic for optic pathways
after primary optic center?
\\\ are axons of 4-th neurons;
\\ begins from ventral posterolateral nucleus of thalamus;
\\\ begins from lateral geniculate body.
\\\ terminates in calcarine fissure.

\\\\ From enumerated, which structure is the most prominent formation of the
extrapyramidal system?
\\\ substantia nigra;
\\ strio-pallidarum system;
\\\ thalamic nuclei;
\\\ corpus Luissi;
\\\ amygdaloid nucleus.

\\\\ From enumerated, what is characteristic for damage of cerebellar vermis?


\\ static ataxia;
\\\ dysmetria;
\\\ “drunken men’s” gait;
\\\ intention tremor.

\\\\ From enumerated, in which posterior horns of the spinal cord the Clark-Shtilling cells
are situated?
\\\ all segments;
\\ only C8 – L2 segments;
\\\ only Th8 – L4 segments.

\\\\ From enumerated, which system directly isn’ t responsible for coordination of
movements?
\\\ vestibular;
\\\ cerebellum;
\\ optic;
\\\ deep sensation.

\\\\ From enumerated, which cranial nerve didn’t carry sensory information from the
face, mouth, ear and mening’s of the brain?
\\\ V cr.n.;
\\\ VII cr.n;
\\ VIII cr.n.;
\\\ IX cr.n.;
\\\ X cr.n.

\\\\ From enumerated, which symptom isn’t observed acording to pathology of VIII
cranial nerve?
\\\ anakusis;
\\ amblyopia;
\\\ hyperakusis;
\\\ surditas.

\\\\ From enumerated, which structure didn’t belong to auditory system?


\\\ external ear;
\\\ middle ear;
\\ lateral ear;
\\\ inner ear.

\\\\ From enumerated, which anatomical feature isn’t characteristic for facial nerve?
\\\ makes outer or external knee;
\\\ makes inner knee;
\\ innervates all mimic muscles of face;
\\\ leaves skull by stylomastoid foramen.

\\\\ From enumerated, what is characteristic for damage of trochlear nerve?


\\ palsy of m. obliquus superior;
\\\ palsy of m. rectus inferior;
\\\ palsy of m. obliquus inferior.

\\\\ From enumerated, which is response for voluntary movements of eyes?


\\\ nuclei of pons Varolli;
\\ area 8 by Brodmann;
\\\ calcarine cortex;
\\\ occipital lobe.

\\\\ From enumerated, were in primary optic center situated center of pupils reaction to
light?
\\\ pulvinar of thalamus;
\\\ lateral geniculate body;
\\ colliculus superior of tectum.

\\\\ From enumerated, which fibers of optic nerves cross through the chiasm?
\\ only nasal half of retina of bouth optic nerves;
\\\ only temporal half of retina of bouth optic nerves;
\\\ temporal half of retina of one optic nerve and nasal half of other.

\\\\ From enumerated, which structure didn’t belong to extrapyramidal system?


\\ internal capsule;
\\\ thalamis;
\\\ reticular formation;
\\\ substantia nigra;
\\\ corpus striatum.

\\\\ From enumerated, which layer of the cerebellum cortex transmits efferent impulses?
\\\ molecular cells;
\\ purkinje cells;
\\\ basket cells;
\\\ granular cells.

\\\\ From enumerated, which nucleus didn’t belong to glossopharyngeal cranial nerve
system?
\\ masticatorius;
\\\ tractus solitari;
\\\ salivatorius;
\\\ ambiguus.

\\\\ From enumerated, which symptom isn’t characteristic for pathology of vestibular
system?
\\\ vestibular ataxia;
\\\ nistagmus;
\\\ vomiting;
\\ dysmetria;
\\\ dizziness.

\\\\ From enumerated, which anatomical feature isn’t characteristic for inner ear?
\\\ cochlea;
\\\ organ of Corti;
\\ Schwalbe’s nucleus;
\\\ acoustic nerve;
\\\ Scarpa‘s ganglion.

From enumerated, what isn’t characteristic for damage of oculomotor nerve?


\\\ ptosis;
\\\ diplopia;
\\ exophtalmos;
\\\ deviation of the eye outwards the nose.
\\\\ From enumerated, which anatomical feature is characteristic for optic tract?
\\\ formed by axons of bipolar neurons;
\\ formed by axons of 3-rd neurons;
\\\ formed by axons of 1-st neurons.

\\\\ From enumerated, what isn’t characteristic for cortical center of I-st cranial nerve?
\\\ double cortical innervation;
\\ controlateral innervation;
\\\ localized in parahippocampal gyrus;
\\\ area 28 by Brodmann.

\\\\ From enumerated, which type of impulsation didn’t enter vermis of cerebellum by
inferior cerebellar peduncle?
\\\ proprioceptive;
\\ exteroceptive;
\\\ vestibular;
\\\ extrapyramidal.

\\\\ From enumerated, what isn’t characteristic for peripheial palsy of XII cranial nerve?
\\\ atrophy of half of the tongue;
\\\ fibrilations of tongue myofibrils;
\\ deviation of tongue from middle line to opposite side of pathologic process.

\\\\ From enumerated, what isn’t characteristic for damage of the accessory nerve?
\\\ difficulty in turning the head to the opposite from process side;
\\\ impossible to raise a hanging shoulder;
\\ to hold up the head;
\\\ impossible to raise the arm to more than 90°.

\\\\ From enumerated, in which case the pseudo-bulbar palsy syndrom is observe?
\\\ bilateral impaurment of cranial nerves IX, X, XI and XII outsude the brainstem;
\\\ bilateral damage of nuclei lower (caudal) cranial nerves in medulla oblongata;
\\ damage of both cortico-nuclear tracts.

\\\\ From enumerated, what didn’t belong to vestibular system?


\\\ labyrinth;
\\\ vestibular nerve;
\\ organ of Corti.

\\\\ From enumerated, which cortical field by Brodmann didn’t take part in process of
analyzing, identifying and comparing the acoustic stimuli?
\\ field 19;
\\\ field 22;
\\\ field 42.

\\\\ From enumerated, which nerve is the subdivision of facial nerve?


\\\ accessory;
\\\ superior laryngeal;
\\ intermediate;
\\\ cochlear.

16. \\\\ From enumerated, in which case the damage of cranial nerve is responsible for
fibrillations and atrophy of half of tongue?
\\\ V cr. n.;
\\\ VII cr. n.;
\\\ VIII cr. n.
\\\ IX cr. n.;
\\\ X cr. n.;
\\ XII cr. n.

\\\\ From enumerated, what is observed in case of impulses full block in the optic nerve?
\\\ amblyopia;
\\\ cortical blindness;
\\\ scotoma;
\\ amaurosis;
\\\ visual hallucinations.
--------------------------------------------------------
\\\\ Motor disfunción is revealed by
\\\ Hyperesthesia
\\\ Hypesthesia
\\\ Paraphasies
\\ Paraparesis

\\\\ Semsory disfunction is revealed by


\\\ Quadriplegia
\\ Dissosiative hypesthesia
\\\ Cerebellar ataxia
\\\ Upper limb paraparesis

\\\\ Acute headache can be caused by


\\ Subarachnoid haemorrage
\\\ Brain tumor
\\\ Hydrocephaly
\\\ Subdural hematoma

\\\\ Acute headache can be caused by


\\ Migraine
\\\ Brain tumor
\\\ Hhydrocephaly
\\\ Subdural hematoma

\\\\ Acute headache can be caused by


\\\ Brain tumor
\\ Meningitis
\\\ Hydrocephaly
\\\ Subdural hematoma

\\\\ Acute headache can be caused by


\\\ Brain tumor
\\ Intracerebral hemorrhage
\\\ Hydrocephaly
\\\ Subdural hematoma

\\\\ Subacute headache can be caused by


\\\ Migraine
\\\ Intracerebral hemorrhage
\\\ Meningitis
\\ Subdural hematoma

\\\\ Subacute headache can be caused by


\\\ Migraine
\\\ Intracerebral hemorrhage
\\\ Meningitis
\\ brain tumor
\\\\ Subacute headache can be caused by
\\\ Migraine
\\\ Intracerebral hemorrhage
\\\ Meningitis
\\ brain absscess

\\\\ Chronic headache can be caused by


\\\ Brain tumor
\\ Posttraumatic Headache
\\\ Hydrocephaly
\\\ Subdural hematoma

\\\\ Chronic headache can be caused by


\\\ Brain tumor
\\ Continued tension headache
\\\ Hydrocephaly
\\\ Subdural hematoma

\\\\ In young patients sudden visual loss is caused by


\\\ Traumatic injury of ocular nerve
\\\ Ishemic neuropathy of ovular nerve
\\\ Tumor of ocular nerve
\\ Optic neuritis

\\\\ In elderly patients sudden visual loss is caused by


\\\ Traumatic injury of ocular nerve
\\ Ishemic neuropathy of ocular nerve
\\\ Tumor of ocular nerve
\\\ Optic neuritis

\\\\ Progressive bilateral visual loss most frequently is caused by


\\\ Bilateral traumatic injury of ocular nerve
\\\ Bilateral ishemic neuropathy of ocular nerve
\\\ Bilateral tumor of ocular nerve
\\ Adenoma of Hypophisis

\\\\ Peripheral cause of vertigo is due to


\\\ Brain stem ishemia
\\\ Brain stem demyelinisation
\\\ Tumor of cerebello-pontine angle
\\ Benign positional paroxysmal vertigo

\\\\ Peripheral cause of vertigo is due to


\\\ Brain stem ishemia
\\\ Brain stem demyelinisation
\\\ Tumor of cerebello-pontine angle
\\ Benign positional paroxysmal vertigo

\\\\ Peripheral cause of vertigo is due to


\\\ Brain stem ishemia
\\\ Brain stem demyelinisation
\\\ Tumor of cerebello-pontine angle
\\ Vestibular neuritis
\\\\ Peripheral cause of vertigo is due to
\\\ Brain stem ishemia
\\\ Brain stem demyelinisation
\\ Meniere disease
\\\ Tumor of cerebello-pontine angle

\\\\ Central vertigo is caused by


\\ Brain stem ishemia
\\\ Meniere disease
\\\ Benign positional paroxysmal vertigo
\\\ Vestibular neuritis

\\\\ Central vertigo is caused by


\\\ Meniere disease
\\\ Benign positional paroxysmal vertigo
\\ Brain stem demyelinisation
\\\ Vestibular neuritis

\\\\ Central vertigo is caused by -


\\\ Meniere disease
\\\ Benign positional paroxysmal vertigo
\\ Brain tumor
\\\ Vestibular neuritis

\\\\ Central vertigo is caused by -


\\\ Meniere disease
\\\ Benign positional paroxysmal vertigo
\\ Brain stem tumor
\\\ Vestibular neuritis

\\\\ In young patients the most frequent cause of spastic paraplegia is-
\\ Multiple sclerosis
\\\ Sensory defecit
\\\ Peripheral motor neuropathy of the legs
\\\ Cervical spondylitis

\\\\ In agied patients the most frequent cause of chronic spastic paraplegia is-
\\\ Multiple sclerosis
\\\ Sensory defecit
\\\ Peripheral motor neuropathy of the legs
\\ Cervical spondylitis
\\\\ In agied patients the most frequent cause of chronic spastic paraplegia is-
\\\ Multiple sclerosis
\\\ Sensory defecit
\\\ Peripheral motor neuropathy of the legs
\\ Metastasis of the spinal cord

\\\\ Sensory ataxia is caused by -


\\ Sensory deficit of the lower extremities
\\\ Cerebellar pathology
\\\ Basal ganglia pathology
\\\ Motor neuropathy of the lower extremities
\\\\ Movement disorders are s characterized by -
\\ Bradykinasia
\\\ Spastic hypertonia and hyperreflexia
\\\ Ataxic gait and hypertonia
\\\ Paraparesis and hypesthesia

\\\\ Movement disorders are s characterized by -


\\\ Spastic hypertonia and hyperreflexia
\\\ Ataxic gait and hypertonia
\\\ Paraparesis and hypesthesia
\\ Excess uncontrolled movemets and hypotonia

\\\ Term hyperkinesis can be applied to -


\\ Tremor
\\\ Ataxia
\\\ Neuropathy
\\\ Quadriplegia

\\\ Term hyperkinesis can be applied to -


\\\ Quadriplegia
\\ Myoclonia
\\\ Ataxia
\\\ Neuropathy

\\\ Term hyperkinesis can be applied to -


\\ Tics
\\\ Ataxia
\\\ Neuropathy
\\\ Quadriplegia
5
\\\ Term hyperkinesis can be applied to -
\\ Torsion dystonia
\\\ Ataxia
\\\ Neuropathy
\\\ Quadriplegia

\\\ Term hyperkinesis can be applied to -


\\\ Ataxia
\\\ Neuropathy
\\\ Quadriplegia
\\ Athethosis
\\\\ Dysfunction of the II cranial nerve is characterized by -
\\\ Hyposmia-anosmia
\\ Ambliopia-amaurosis
\\\ Hypacusia-anacusis
\\\ Hypogevsia-agausia

\\\\ Dysfunction of the I cranial nerve is characterized by -


\\ Hyposmia-anosmia
\\\ Ambliopia-amaurosis
\\\ Hypacusia-anacusis
\\\ Hypogevsia-agausia
\\\\ Dysfunction of the VIII cranial nerve is characterized by -
\\\ Hyposmia-anosmia
\\\ Ambyopia-amaurosis
\\ Hypacusia-anacusis
\\\ Hypogevsia-agausia

\\\\ Dysfunction of the VIII cranial nerve is characterized by -


\\\ Hyposmia-anosmia
\\\ Ambyopia-amaurosis
\\ Vertigo -tinnitus
\\\ Hypogevsia-agausia

\\\\ Dysfunction of the VIII cranial nerve is characterized by


\\\ Hyposmia-anosmia
\\\ Ambyopia-amaurosis
\\ Vertigo -tinnitus
\\\ Hypogevsia-agausia

\\\\ Dysfunction of the IX-X cranial nerve is characterized by


\\\ Hyposmia-anosmia
\\\ Ambyopia-amaurosis
\\\ Vertigo -tinnitus
\\ Hypogevsia-agausia

\\\\ Dysfunction of the IX-X cranial nerve is characterized by -


\\\ Hyposmia-anosmia
\\\ Ambyopia-amaurosis
\\\ Vertigo -tinnitus
\\ Dysphagia-Dysphonia

\\\\ Partial loss of visual field is called -


\\ Hemianopsia
\\\ Amaurosis
\\\ Ambliopia
\\\ Dischromatophsia

\\\\ Deficit of color perception is called-


\\\ Hemianopsia
\\\ Aamaurosis
\\\ Ambliopia
\\ Dischromatopsia

\\\\ Dysfunction of the III-IV-VI cranial nerves is characterized by all symptoms except
\\\ Diplopia, phtosis
\\\ Myosis,
\\\ Midriasis
\\ Ambliopia- amaurosis

\\\\ Dysfunction of the III-IV-VI cranial nerves is characterized by all symptoms except
\\\ Diplopia, phtosis
\\\ Myosis,
\\\ Horner symptom
\\ Ambliopia- amaurosis
\\\\ Pathology of pupils is characterised by -
\\ Anisokoria
\\\ Phtosis
\\\ Homonymos hemianophsia
\\\ Bitemporal hemianophsia

\\\\ Homonymos hemianophsia is characterised by -


\\ Amaurosis in the same sides of visual fields of both eyes
\\\ Amaurosis in the different sides of visual fields of both eyes
\\\ Cortical blindness
\\\ Bilaterla amaurosis

\\\\ Heteronymos hemianophsia is characterised by -


\\\ Amaurosis in the same sides of visual fields of both eyes
\\ Amaurosis in the different sides of visual fields of both eyes
\\\ Cortical blindness
\\\ Bilaterla amaurosis

\\\\ Altered consciousness is defined as -


\\\ Dementia
\\\ Sleep
\\ Delirium
\\\ Vertigo

\\\\ The single form of altered consciousness is --------------


\\\ Confusional state
\\ sleep
\\\ Lethargy
\\\ Catatonic stupor

\\\\ Which condition do not correspond to the term “altered conciousness?


\\\ Confusion, delirium
\\\ stupor, Coma
\\ Panic attack
\\\ Lethargy

\\\\ The deepest level of altered concsiousness is


\\\ Stupor
\\\ Syncope
\\ Coma
\\\ Lethargy

\\\\ Which statement is correct -


\\\ Coma is caused by damage at the level of medulla oblongata
\\\ In comatose state it is neseccary to have motor activity
\\\ Glasgow coma scale evaluate only depth of coma.
\\ Score 8-4 at Glasgow Coma Scale corresponds with coma.

\\\\ Which score of Glasgow Coma Scale corresponds with coma


\\\ Score 15-14
\\ Score 8-4
\\\ Score 13-9
\\\ Score 3
\\\\ Among listed below conditions, focal cause of coma state can be attributed to
\\\ Hyperglicemia
\\\ Hepatargia
\\\ Mixedema
\\ Epidural hematoma

\\\\ Among listed below conditions, diffusel cause of coma state can be attributed to
\\\ Epidural hematoma
\\\ Intracerebral hemorrhage
\\ Encephalitis
\\\ Cerebellar infarction

\\\\ Among listed below conditions, subtentorial cause of coma state can be attributed to -
\\\ Anterior cerebral artery infarcion
\\\ Hypoglicemia
\\\ Lobar hemorrhage
\\ Cerebellar infarction

\\\\ Development o coma in the case of subtentorial mass lesion is caused by


\\ Medular dislocation
\\\ Hhorizontal dislocation
\\\ Diffuse systemic damage of the brain
\\\ Transtentorial dislocation.

\\\\ Urgent measures for comatose patient include -


\\ Permeability of airway system (A), adequate breathing (B), stable circulation (C)
\\\ Neurovizualisation
\\\ Lumbalur puncture
\\\ Correction of hypotheermy

\\\\ Urgent measures for comatose patient include


\\ Correction of possible hypoglicemia
\\\ Neurovizualisation
\\\ Detailed neurological examination
\\\ Correction of hypotheermy

\\\\ Coma which is preceeded by delirium and where focal neurological symptoms are
absent indicates on -
\\ Metabolic coma
\\\ Brain tumor
\\\ Brain stem stroke
\\\ subarachnoid hemorrhage

\\\\ Coma which develops during several days or weeks and is accompanied by focal
neurological signs indicates on presence of -
\\ Chronic subdural hematoma
\\\ Intracerebral hematoma
\\\ Hypoglicemiae
\\\ Ssubarachnoid hemorrhage

\\\\ EEG is of superior importance in the diagnose of -


\\\ Parkinson disease.
\\\ Multiple sclerosis
\\ Epilepsy
\\\ Comatose state

\\\\ ENMG is of superior importance in the diagnose of


\\\ Parkinson disease.
\\\ Multiple sclerosis
\\ Distal neuropathy
\\\ Comatose state

\\\\ When focal signs of brain damage are absent, coma can be caused by -
\\\ Ischemic strokei
\\ Encephalitis
\\\ Hemorrhagic stroke
\\\ Cerebellar stroke

\\\\ In comatose state EEG data of slow wave activity indicate presence of
\\\ Hepatargia
\\\ Subarachnoid hemorrhage
\\\ Hypoglicemia
\\ Encephalitis

\\\\ Coma, caused by focal brain damage can be due to


\\\ Hepatargia
\\\ Subarachnoid hemorrhage
\\\ meningitis
\\ Subdural and epidural hematoma

\\\\ Coma, caused by supratentorial damage can be due to -


\\\ Tumor of frontal pole
\\\ tumor of the temporal lobe
\\\ Brain stem hemorrhage
\\ Subdural and epidural hematoma

\\\\ Coma, caused by infratentorial damage can be due to


\\\ Tumor of frontal pole
\\\ tumor of the temporal lobe
\\ Brain stem hemorrhage
\\\ Ishemic stroke of occipital lobe

\\\\ From listed below statements correct is that- ------------


\\\ Locked in syndrome is caused by damage of diencephalon
\\\ Locked in syndrome can bes caused by pontine hemorrhage
\\\ Locked in syndrome is accompanied by altered consiousness
\\ In Locked in syndrome horizontal eye movements are preserved

\\\\ From listed below statements criteria of brain death is


\\\ Glasgow coma scale score 6
\\\ Presence of oculovestibular and oculocephalic reflexes
\\\ Myotic and areactive pupils
\\ Glasgow coma scale score 3
\\\\ From listed below statements criteria of brain death is -
\\ Glasgow coma scale score 3
\\\ Hypotension
\\\ Hypothermia
\\\ In coma caused by barbiturate intoxication isoelectric EEG pattern

\\\\ Clinical symptoms of hydrocephaly consists of -


\\\ VII cranial nerve palsy
\\ Lower limb paraparesis and ataxia
\\\ Down gaze paresis
\\\ Upper limb par paresis

\\\\ Clinical symptoms of hydrocephaly consists of -


\\\ Hemiparesis
\\\ Bulbar syndrome
\\ Upper gaze palsy
\\\ III cranial nerve palsy

\\\\ Clinical symptoms of normotensive hydrocephaly consists of -


\\ Cognitive decline
\\\ Extrapyramidal syndrome
\\\ Complex partial seizures
\\\ Dissociative sensory disturbances

\\\\ Hydrocephaly can be treated by


\\ Ventriculoperitoneal shunting
\\\ Enlargement of foramen Monro
\\\ Resection of cerebellar tonsiles
\\\ Ventricular drainage.

\\\\ Benign intracranial Hypertension can be caused -


\\\ Endocrine pathology
\\\ Bronchial asthma
\\\ Thyreotoxicosis
\\ Gastric ulcer

\\\\ In embriogenesis stage of primary neurulation includes -


\\\ 1-2 month gestation
\\\ 4-7 week of gestation
\\\ 3-5 month of gestation
\\ 3-4 week of gestation

\\\\ In embriogenesis stage of secondary neurulation includes -


\\\ 1-2 month gestation
\\ 4-7 week of gestation
\\\ 3-5 month of gestation
\\\ 3-4 week of gestation

\\\\ In embriogenesis stage of migration includes -


\\\ 1-2 month gestation
\\\ 4-7 month of gestation
\\ 3-5 month of gestation
\\\ 3-4 week of gestation
\\\\ In embriogenesis stage of proliferation includes -
\\\ 1-2 month gestation
\\\ 2-4 month of gestation
\\\ 3-5 month of gestation
\\ 3-4 week of gestation

\\\\ In embriogenesis stage of myelinisation includes -


\\\ 1-2 month gestation
\\\ 2-4 month of gestation
\\\ 3-5 month of gestation
\\ develops at postnatal stage

\\\\ Disorder of primary neurulation includes -


\\\ Vascular malformations of the foetus
\\\ Cerebral malformations
\\ Anencephaly
\\\ Scoliosis.

\\\\ Disorder of primary neurulation includes -


\\\ Vascular malformations of the foetus
\\\ Cerebral malformations
\\ Encephalocele
\\\ Scoliosis.

\\\\ Disorder of primary neurulation includes -


\\\ Vascular malformations of the foetus
\\\ Cerebral malformations
\\\ Scoliosis.
\\ Myelomeningocele

\\\\ Term cerebral palsy supposes -


\\\ Amaurosis and anacusis developped in early life.
\\\ Astasia-abasia devellopped in early life
\\\ Autonomic failure developped in early life
\\ Disorders of movement and posture developped in early life

\\\\ Important disorder of migration is -


\\\ Phakomatosis
\\\ Cerebral palsy
\\ Lissencephaly and Schizencephaly
\\\ Chiari malformation

\\\\ Lissenencephaly is the term used to describe –


\\\ Grey matter lined clefts that extend through the cerebral hemisphere from the
ependymal lining of lateral ventricles to the pial lining of the cortex
\\\ Cortical dysplasia where neurons reach the cortex but distribute abnormally
\\ Smooth brain with lack of gyraland sulcal development on the surface of the brain.
\\\ Displacement of cerebellas tonsilles to the occipital foramen

\\\\ Schizencephaly is the term used to describe –


\\ Grey matter lined clefts that extend through the cerebral hemisphere from the
ependymal lining of lateral ventricles to the pial lining of the cortex
\\\ Cortical dysplasia where neurons reach the cortex but distribute abnormally
\\\ Smooth brain with lack of gyral and sulcal development on the surface of the brain.
\\\ Displacement of cerebellas tonsilles to the occipital foramen

\\\\ Among listed below dysraphic syndrome is


\\\ Lissencephaly
\\\ Schizencefalia
\\ Diastematomyelia
\\\ Polimikrogiria

\\\\ Syringomielia is defined as


\\\ Developmental anomaly of spinal sensory radicles
\\\ Developmental anomaly of spinal motor radicles
\\ Formation of cavities in the spinal cord
\\\ Stenosis of spinal cord

\\\\ Phacomatosis are-


\\\ Combined mesoectodermal developmental anomaly that is characterised by
genetically determioned dysfunction of vascular, nervous systems and skin.
\\ Combined neuroectodermal developmental anomaly that is characterised by
genetically determioned dysfunction of nervous system, eyes and skin.
\\\ Combined mesoectodermal developmental anomaly that is characterised by
genetically determioned dysfunction of gastrointestinal, cardiovascular and nervous
systems.
\\\ Combined mesoectodermal developmental anomaly that is characterised by
genetically determioned dysfunction of cranial nerves, vascular system and skin.

\\\\ Neurofibromatosis is characterised by -


\\\ Hypopigmental areas and butterfly like redish-brown nodules – adenoma sebaceum
\\ Caffe-au lait patches and neurofibromas across the nerves
\\\ Radish-blue naevus, involving distribution of the I root of the trigeminal nerve .
\\\ Multiple hemangioblastomas at the retina

\\\\ Hippel-Lindau disease is characterised by -


\\\ Hypopigmental areas and butterfly like redish-brown nodules – adenoma sebaceum
\\\ Caffe-au lait patches and neurofibromas across the nerves
\\\ Radish-blue naevus, involving distribution of the I root of the trigeminal nerve .
\\ Multiple hemangioblastomas at the retina

\\\\ Tuberous sclerosis is characterised by -


\\ Hypopigmental areas and butterfly like redish-brown nodules – adenoma sebaceum
\\\ Caffe-au lait patches and neurofibromas across the nerves
\\\ Radish-blue naevus, involving distribution of the I root of the trigeminal nerve .
\\\ Multiple hemangioblastomas at the retina
\\\\ Encephalo-facial angiomatosis is characterised by -
\\\ Hypopigmental areas and butterfly like redish-brown nodules – adenoma sebaceum
\\\ Caffe-au lait patches and neurofibromas across the nerves
\\ Radish-blue naevus, involving distribution of the I root of the trigeminal nerve .
\\\ Multiple hemangioblastomas at the retina
\\\\ Attention deficit hyperactivityu disease (ADHD)
\\\ Coexistance of motor, sensor and extrapyramidal pathology
\\ Coexistance of neurological, cognitive and behavioral disturbances
\\\ Coexistance of cerebellar, cognitive and paroxusmal disturbances.
\\\ Coexistance of visual, auditory and cognitive disturbances.

\\\\ For demyelinating diseases pathological hallmark is


\\\ Appearance of calcification in grey matter.
\\\ Damage of cell membranes.
\\ Damage of myelin sheath .
\\\ Prolifération of mesoglia in white matter of the brain.

\\\\ Demyelinisation can be caused by -


\\\ Ageing of the nervous system.
\\\ Tymor of nerve fibers
\\ Immune-mediated pathology
\\\ Vascular pathology

\\\\ In dismyelinating pathology destroying of myelin sheath is due to -


\\\ Traumatic injury
\\\ Ishemic pathology
\\ Genetic deficit
\\\ Immune-mediated pathology

\\\\ Myelin of CNS is formed by -


\\\ Astrocytes
\\ Oligodendroglial cells
\\\ Microglial cells
\\\ Ependimal cells

\\\\ From listed below diseases demyelinating pathology is -


\\\ Hemorrhagic strokei
\\\ eepilepsy
\\ Acute hemorrhagic leucoencephalopathy
\\\ Migraine
Multiple Sclerosis
\\\\ For multiple sclerosis it is typical presence of
\\\ Focal seizures
\\\ Meningeal signs
\\\ High temperature
\\ Remmitting attacks of focal or multifocal neurological disfunction.

\\\\ Multiple sclerosis is frequent -


\\\ In newborns
\\\ At the age 10-15 years
\\ At the age 20-40 years
\\\ In aged persons ( older than 70)

\\\\ Risk of multiple sclerosis is high in those persons who -


\\\ Had myocardial infarction
\\ Had infectious diseases of childhood at the age of 12-15 years.
\\\ Have diabettes mellitus
\\\ Had severe traumatic injury of the brain
\\\\ The main cause of multiple sclerosis is -
\\ Perivascular infiltration of lymphocytes and phagocytose of myelin.
\\\ Dysfunction of strio-nigral tract
\\\ Dysfunction of membrane ion channels
\\\ Dysfunction in Copper metabolism

\\\\ In patients with multiple sclerosis stable neurological deficit is due to -


\\ Axonal damage
\\\ Eosinophilic pleocytosis
\\\ Disturbance between T-kiliers and T helpers ratio
\\\ Periventricular demyelinisation

\\\\ In patients with multiple sclerosis most commonly seen sign is –


\\\ Lasseg sign
\\ Lhermitte sign
\\\ Kerning sign
\\\ Bruzinski sign

\\\\ New attack of multiple sclerosis can be defined if time between onset of two attacks is not less than -
\\\ 10 days
\\\ 15 days
\\\ 20 days
\\ 30 days.

\\\\ Frequency of attacks of multiple sclerosis is defined by their number during


\\\ 6 month
\\\ 8 month
\\\ 10 month
\\ 12 month

\\\\ Relapse of multiple sclerosis is identified as acute neurological defecit lasting not less than -
\\\ 2 hours
\\\ 10 hours
\\\ 4 hours
\\ 24 hours

\\\\ MRI diagnostin criteria of multiple sclerosis are determined as


\\\ 1 or two demyelinating lesions
\\\ Lesions more than 2 mm in diameter
\\ one or more lesion in periventricular space, subtentorial area or corpus callosum
\\\ Round or ovale shaped of lesion of less than 1mm in diameter .
Myelit
\\\\ In acute myelitis -
\\ Ttransverse section of spinal cord revealed damage of all spinal tracts
\\\ Signs of spinal cord damage are asymmetric
\\\ Damage of spinal cord destroy onle several tracts
\\\ Demyelinating areas extend to the pons and cerebellum.

\\\\ Acute myelitis is characterized by -


\\\ Hemiplegy and hemiparesis
\\\ In the trunk segmental loss of sensation
\\ Disturbance of deep sensation in lower extremities
\\\ Retrobulbar neuritis.
\\\\ Recurrent myelitis transversa can be due to -
\\ Lupus erythematosus
\\\ Trigeminal neuralgia
\\\ West syndrome.
\\\ Torsion dystonia

\\\\ Acute disseminating encephalomyelitis is treated by


\\ Corticostroids
\\\ Anticonvulsants
\\\ Immune stimulating drugs
\\\ Vasoactive drugs.

\\\\ For myelitis transversa it is typical presence of -


\\ Paraplegia and anesthesia
\\\ Hemiplegia and hemianesthesia
\\\ Alternatrive syndroms
\\\ Dissociative sensory disturbances.
Headache
\\\\ Promary headache include -
\\\ Headache caused by subdural hematoma
\\\ Headache caused by arteriitis
\\ Cluster headache
\\\ Headache caused by arterial dissection

\\\\ Diagnostic criteria of migraine include –


\\\ Bilateral headache filling as like a band
\\\ Preserved phisical activity
\\ Nausea and vomiting
\\\ Absence of photo-phonophobia

\\\\ Diagnostic criteria of migraine include –


\\\ Bilateral headache filling as like a band
\\\ Preserved phisical activity
\\\ Absence of nausea and vomiting
\\ Pulsating headache

\\\\ Diagnostic criteria of migraine include


\\\ Bilateral headache filling as like a band
\\\ Preserved phisical activity
\\\ Absence of nausea and vomiting
\\ Pulsating unilateral headache

\\\\ Migraine attacks generally starts -


\\\ At puberty
\\\ At the age 5-10 years old .
\\ At the age 20-30 years old
\\\ After 50years old age

\\\\ Migraine attacks lasts for


\\\ 1/2-1 hour
\\\ 2-3 hours
\\ 4-72 hours
\\\ 15 minutes
\\\\ Migraine auralasts for -
\\ >5<60 minutes
\\\ 60-90 minutesi
\\\ 90-120 minutes
\\\ 120-160 minutes

\\\\ Specific drugs for treatment of migraine are


\\\ Vasoactive drugs
\\ Triptans
\\\ Nootrops.
\\\ Benzodiasepines .

\\\\ With ageing frequency of migraine attacks -


\\\ Increases
\\ Decreases
\\\ Does not change
\\\ Depends upon sex

\\\\ Trigeminal neuralgia is charaqcterised by -


\\\ Localised headache of the frontal region
\\ Acute burning pain in the area of trigenimal nerve roots .
\\\ Pulsatile unilateral headache .
\\\ Dull headache that appeared only at night.

\\\\ For the treatment of trigeminal neuralgia are used -


\\ Anticonvulsants
\\\ Neuroleptics
\\\ Beta-blockers
\\\ Calci – antagonists .

\\\\ Cluster headache


\\\ Is bilateral
\\\ Changes side .
\\ Always appears at the same side
\\\ Appears in the occipital region

\\\\ Appearance of blood in CSF is typical for -


\\\ Meningitis
\\ Subarachnoid hemorrhage
\\\ Status epilepticus
\\\ Ishemic stroke

\\\\ Gold standard for diagnosis of subarachnoid hemorrhage is


\\ CT of the brain
\\\ Lumbar puncture
\\\ MRI of the brain
\\\ Ultrasound dopplerography of the brain vessels

\\\\ Gold standard for diagnosis of saccular aneurysm is -


\\\ CT of the brain
\\ Cerebral angiography
\\\ MRI of the brain
\\\ Ultrasound dopplerography of the vessels
\\\\ Cerebral vasospasm developped after the subarachnoid hemorrhage can be
detected by -
\\\ CT
\\\ Lumbar puncure
\\\ MRI
\\ Ultrasound dopplerography of the cerebral vessels

\\\\ Arterial aneurysms of cerebral vessels are treated by -


\\\ Conservative treatment
\\\ Baloon angioplasty
\\ Aneurysl clipping and endovascular embolisation
\\\ Endarterectomy

\\\\ Gold standard for diagnosis of cerebrovascular malformation is -


\\\ CT of the brain
\\\ Cerebral angiography
\\ MRI of the brain
\\\ Ultrasound dopplerography of the vessels

\\\\ The most frequent neurological disease in adults is -


\\\ Epilepsy
\\\ Tumors of the brain
\\ Cerebrovascular diseases
\\\ Traumatic injury of the brain

\\\\ Incidence of stroke is -


\\\ 100/100 000 of population
\\ 200/100 000 of population
\\\ 50/100 000 of population
\\\ 300/100 000 of population

\\\\ Vertebral artery is division of –


\\\ Arteria carotis externa
\\\ Arteria carotis interna
\\ Arteria subclavia
\\\ Aorta

\\\\ In patient with minor stroke -


\\\ Clinical signs disappear in 24 hours
\\ Clinical signs disappear in 7 days
\\\ Clinical signs are stable for an year
\\\ Clinical signs are stable for a long time

\\\\ In patient with transient ischemic attack -


\\ Clinical signs disappear in 24 hours
\\\ Clinical signs disappear in 7 days
\\\ Clinical signs are stable for an year
\\\ Clinical signs are stable for a long time
\\\\ Stroke of the anterior cerebral artery cause -
\\ Sensomotor l hemiparesis with prevalence of weakness in the leg
\\\ Contralateral hemiplegy, hemisensory deficit, homonymous hemianopsia , aphasia or
gnostic disrturbances
\\\ Syndrome of Wallenberg
\\\ Tetraplegy, damage of cranial nerves, altered conscious

\\\\ Stroke of the middle cerebral artery cause -


\\\ Sensomotor l hemiparesis with prevalence of weakness in the leg
\\ Contralateral hemiplegy, hemisensory deficit, homonymous hemianopsia , aphasia or
gnostic disrturbances
\\\ Syndrome of Wallenberg
\\\ Tetraplegy, damage of cranial nerves, altered conscious

\\\\ Stroke of the inferior posterior cerebellar l artery cause -


\\\ Sensomotor l hemiparesis with prevalence of weakness in the leg
\\\ Contralateral hemiplegy, hemisensory deficit, homonymous hemianopsia , aphasia or
gnostic disrturbances
\\ Syndrome of Wallenberg
\\\ Tetraplegy, damage of cranial nerves, altered conscious

\\\\ Stroke of the basilar artery cause -


\\\ Sensomotor l hemiparesis with prevalence of weakness in the leg
\\\ Contralateral hemiplegy, hemisensory deficit, homonymous hemianopsia , aphasia or
gnostic disrturbances
\\\ Syndrome of Wallenberg
\\ Tetraplegy, damage of cranial nerves, altered conscious

\\\\ Pure motor stroke involve -


\\\ Only hemisensory disturbances
\\ Isolated hemiparesis without other signs
\\\ Coexistance of hemiparesis and hemisenory disturbances without other signs
\\\ Paresis of the leg and ipsilateral ataxia

\\\\ Pure sensory stroke involve -


\\ Only hemisensory disturbances
\\\ Isolated hemiparesis without other signs
\\\ Coexistance of hemiparesis and hemisenory disturbances without other signs
\\\ Paresis of the leg and ipsilateral ataxia

\\\\ Sensormotor motor involve -


\\\ Only hemisensory disturbances
\\\ Isolated hemiparesis without other signs
\\ Coexistance of hemiparesis and hemisenory disturbances without other signs
\\\ Paresis of the leg and ipsilateral ataxia

\\\\ Ataxic hemiparesis involve -


\\\ Only hemisensory disturbances
\\\ Isolated hemiparesis without other signs
\\ Coexistance of hemiparesis and hemisenory disturbances, without other signs
\\ Paresis of the leg and ipsilateral ataxia

\\\\ Pure motor hemiparesis involve -


\\\ Only hemisensory disturbances
\\ Isolated hemiparesis without other signs
\\\ Coexistance of hemiparesis and hemisenory disturbances without other signs
\\\ Paresis of the leg and ipsilateral ataxia

\\\\ From listed below causes, most frequently stroke is due to -


\\\ Cardiogenic emboli
\\ Big artery atherothrombosis and thromboembolism
\\\ Small vessel disease
\\\ Arteriitis and arterial dissection

\\\\ From listed below causes, very seldom stroke is due to -


\\\ Cardiogenic emboli
\\\ Big artery atherothrombosis and thromboembolism
\\\ Small vessel disease
\\ Arteriitis and arterial dissection

\\\\ A pure motor stroke is most likely with damage to the -


\\ Internal capsule
\\\ Cerebellum
\\\ Putamen
\\\ Caudate

\\\\ A pure sensory stroke is most likely with damage to the


\\\ Internal capsule
\\ Thalamus
\\\ Hippocampus
\\\ Globus pallidus

\\\\ Occlusion of which of the following arteries typically produces lateral medullary
infarction syndrome? –
\\\ Basilar artery
\\ Vertebral artery
\\\ Superior cerebellar artery
\\\ Anterior inferior cerebellar artery(AICA)

\\\\ A 72-year-old woman has the abrupt onset of the right face and hand weakness,
disturbed speech production and a right homonymous hemianopsia. This is most likely
attributable to occlusion of the -
\\ Left middle cerebral artery
\\\ Left anterior cerebral artery
\\\ Left vertebrobasilar artery
\\\ Right anterior choroidal artery

\\\\ A39-year-old woman has diplopia several times a day for 6 weeks She consults a
physician when the double vision becomes unremitting, and also complains of dull pain
behind her right eye. When a red glass is placed over her right eye and she is asked to
look at a flashlight off to her left, she reports seeng a white light and a red light. The red
light appears to her to be more to the left than the white light . Her right pupil is more
dilated tha her left pupil and and responds less briskly to a bright light directed at it than
does the left pupil.Before any further investigations can be performed, the woman
develops the worst headache in her life and becomes stuporous. Her physician
discovers that she has marked neck stiffness and photophobia. The pfysician performs
a transfemoral angiogram. The radiologic study is expected toreveal that
the woman has
\\\ An arterionenous malformation
\\\ An occipital astrocytoma
\\\ A sphenoidal meningeoma
\\ A saccular aneurism

\\\\ A 39-year-old woman has diplopia several times a day for 6 weeks She consults physician
when the double vision becomes unremitting, and also complains of dull pain behind her right
eye. When a red glass is placed over her right eye and she is asked to look at a flashlight off
to her left, she reports seeng a white light and a red light. The red light appears to her to be
more to the left than the white light . Her right pupil is more dilated tha her left pupil and and
responds less briskly to a bright light directed at it than does the left pupil. The cranial nerve
injury likely to be responsible for all these observations is one involving -
\\\ The second cranial nerve
\\ The third cranial nerve
\\\ The fourth cranial nerve
\\\ The sixth cranial nerve

\\\\ A 39-year-old woman has diplopia several times a day for 6 weeks She consults
physician when the double vision becomes unremitting, and also complains of dull pain
behind her right eye. When a red glass is placed over her right eye and she is asked to
look at a flashlight off to her left, she reports seeing a white light and a red light. The red
light appears to her be more to the left than the white light . Her right pupil is more
dilated tha her left pupil and and responds less briskly to a bright light directed at it than
does the left pupil. The site of the lesion responsible for this woman’s symptoms and
signs is most probably the
\\\ Anterior communicating artery
\\ Posterior communicating artery
\\\ Anterior cerebral artery
\\\ Middle cerebral artery

\\\\ A 39-year-old woman has diplopia several times a day for 6 weeks She consults
physician when the double vision becomes unremitting, and also complains of dull pain
behind her right eye. When a red glass is placed over her right eye and she is asked to
look at a flashlight off to her left, she reports seeng a white light and a red light. The red
light appears to her to be more to the left than the white light . Her right pupil is more
dilated tha her left pupil and and responds less briskly to a bright light directed at it than
does the left pupil. Three days after developing stiffness and photophobia, the woman
develops left-sided weakness and hyperreflexia. her left plantar response is upgoing.
Her physician presumes that these deficits are a delaye defect of the subarachnoid
blood and so would treat her with
\\\ heparin
\\\ warfarim
\\ nimodipine
\\\ carbamazepine

\\\\ A 73-year –old man with a history of hypertension complains of a 10-min episodes of
leftsided weakness and slurred speech. On further questioning he relates three brief
episodes in the last month of sudden impairment of vision affecting the right eye. His
examination now is normal. Which of the followingwould be the most appropriate next
diagnostic test? -
\\\ Creatine phosphokinase
\\\ Holter monitor
\\\ Visual evoked potentials
\\ Carotid artery Doppler ultrasound

\\\\ A 73-year –old man with a history of hypertension complains of a 10-min episodes of
leftsided weakness and slurred speech. On further questioning he relates three brief
episodes in the last month of sudden impairment of vision affecting the right eye. His
examination now is normal. The episodes of visual loss are most likely related to -
\\\ Retinal vein thrombosis
\\ Central retinal artery ischemia
\\\ Posterior cerebral artery ischemia
\\\ Middle cerebral artery ischemia

\\\\ A 73-year –old man with a history of hypertension complains of a 10-min episodes of
leftsided weakness and slurred speech. On further questioning he relates three brief
episodes in the last month of sudden impairment of vision affecting the right eye. His
examination now is normal. A thorough evaluation reveals that patient has a 90%
stenosis of the right internal carotid artery at the bifurcation. The management option
most likely to prevent a future stroke is which of the following?-
\\\ Warfarin
\\\ Carotid artery angioplasty
\\ Carotid endarterectomy
\\\ Extracranial-intracranial bypass

\\\\ A 62-year old man with a history of myocardial infarction awakens with a dense right
sided hemiplegia. His eyes are tonically deviated to the leftand he doesnot respond to
threat on the right side of his visual field. He appears to be alert and resonds to pain on
the left side of his body. His speech is unintelligible and nonfluent and he follows no
instructions. Efforts to get him to repeat simple pfrases cocsistently fail.Pick the
language disturbance that best explaines the clinical picture -
\\\ Broca’s aphasia
\\\ Wernicke’s aphasia
\\ Global aphasia
\\\ Conduction aphasia

\\\\ A 45-year-old woman with chronic atrial fibrillation discontinues warfarin treatment
and abruptly develops problems with language comprehension. She is able to produce
some intelligible phrases and produces sound quite fluently; however, she is unable to
follow simple instructions or to repeat simple phrases On attempting to write, she
becomes very frustrated and agitated. Emergency MRI reveals a lesion of the left
temporal lobe, that extends into the superior temporal gyrus. Pick the language
disturbance that best explaines the clinical picture-
\\\ Broca’s aphasia
\\\ Wernicke’s aphasia
\\ Transcortical sensory aphasia
\\\ Anomic aphasia

\\\\ A 71-year-old man develops headache and slight difficulty speaking while having
sexual intercourse. He has a long-standing history of hypertension, but has been on
medication for more than 7 years. He makes frequent errprs in finding words and
follows complex commands somewhat inconsistently. The most obvious dfect in his
language function is his inability to repeat the simplest of phrases without making
repeated errors. An emergency CT scan reveals an intracerebral hemrrahage in the left
parietal lobe that appears to communicate with the lateral ventricle. Pick the language
disturbance that best explaines the clinical picture -
\\\ Broca’s aphasia
\\\ Wernicke’s aphasia
\\\ Anomic aphasia
\\ Conduction aphasia

\\\\ A 24-year-old woman abruptly loses all speech during the third trimester of otherwise
uncomplicated pregnancy. She has a history of severe migraines during which she
occasionally develops a transient right hemisplegia.Her comprehension is good, and
she is frustrated by her inability to speak or write. She is unable to repeat simple
phrases, but she does begin to produce simple words within 5 days of the acute
disturbance of language. Pick the language disturbance that best explaines the clinical
picture -
\\ Broca’s aphasia
\\\ Wernicke’s aphasia
\\\ Transcortical sensory aphasia
\\\ Transcortical motor aphasia

\\\\ A 78-year-old man sufferrs a cardiac arrest while being treated in an emergency
room for chest pain. Resuscitation is initiated immediately, but profound hypotension is
observed for at least 20 minutes. A cardiac rhythm is restored, but the patienr remains
unconscious for the next 3 days. When he is awake, alert and extubated, his speech is
limited to repetition of words and sounds produced by those around him. He has no
apparent comprehention of language and produces few sounds spontaneously.
Whenever the patient is spoken to, he fairly accurately repeats what was said to
him.Pick the language disturbance that best explaines the clinical picture
\\\ Transcortical motor aphasia
\\\ Anomic aphasia
\\\ Global aphasia
\\ Mixed transcortical apasia

\\\\ A 75-year old man with a history of recent memory impairment is admitted with
headache, confusion, and a left homonymous hemianopsia. He has recently had two
episodes brief unresponsiveness. Tere is no history of hypertension. Computed
tomography (CT) scan shows a right occipital lobe hemorrhage with some subarachnoid
extension of the blood. An MRI scan with gradient echo sequence reveals foci of
hemosiderin in the right temporal and left frontal cortex. The likely cause of this patient’s
symptoms and signs is
\\\ Gliomatosis cerebri
\\\ Multi-infarct-dementia
\\\ Mycoic aneurism
\\ Amyloid angiopathy

\\\\ A 61year-old man with a history of hypertension has been in exellent health until
presents with vertigo and , dunsteadyness lasting for two days. He then developes
nausea, vomiting, dysphagia, hoarseness, ataxia, left facial pain, and right-sided
sensory loss. There is no weakness. On examination, he is alert, with a normal mental
status. He vomits with head movement. There is skew deviation of the eyes, left ptosis,
clumsiness of the left arm,and titubation. He has loss of pain and temperature sesation
in the left foot. He is unable to walk. Magnetic resonance imaging(MRI) in this patient
mighr be expected to show which of the following? -
\\\ Basilar artery tip aneurysm
\\\ Right lateral medullary infarction
\\ Left lateral medullary infarction
\\\ Left medial medullary infarction
\\\\ A 61year-old man with a history of hypertension has been in excellent health until he
presents with vertigo and , dunsteadyness lasting for two days. He then develops
nausea, vomiting, dysphagia, hoarseness, ataxia, left facial pain, and right-sided
sensory loss. There is no weakness. On examination, he is alert, with a normal mental
status. He vomits with head movement. There is skew deviation of the eyes, left ptosis,
clumsiness of the left arm,and titubation. He has loss of pain and temperature sensation
in the left foot. He is unable to walk. The dysphagia in this case is secondary to
involvement of which of the following structures? -
\\\ Nucleus silitarius
\\\ Nucleus and descending tract of CNV5
\\ Nucleus ambiguus
\\\ Lateral spinothalamic tract

\\\\ A 61year-old man with a history of hypertension has been in exellent health until
presents with vertigo and , dunsteadyness lasting for two days. He then developes
nausea, vomiting, dysphagia, hoarseness, ataxia, left facial pain, and right-sided
sensory loss. There is no weakness. On examination, he is alert, with a normal mental
status. He vomits with head movement. There is skew deviation of the eyes, left ptosis,
clumsiness of the left arm,and titubation. He has loss of pain and temperature sesation
in the left foot. He is unable to walk.Right sided sensory loss is secondary to
involvement of which of the following structures? -
\\\ The right lateral spinothalamic tract
\\ The left.lateral spinothalamic tract
\\\ Nucleus and descending tract of CNV5
\\\ Ninth and Tenth cranial nerves

\\\\ The most striking neurologic complication of von Economos encephalitis


(encephalitis lethargica), a type of encephalitis that occurred in epidemic proportions
along with viral influenca between 1917 and 1928 was -
\\\ Blindness
\\\ Hearing loss
\\\ Paraplegia
\\ Parkinsonism

\\\\ A 17-year old female presents initially with fever and progressive weakness. An
extensive
neurological evaluation including EMG suggests a motor neuron disease. The motor
neuron
disease most certainly trased to a virus is -
\\ Poliomyelitis
\\\ Subacute sclerosing panencephalitis
\\\ Progressive multifocal leukoencephalopathy(PML)

\\\ Subacute encephalomyelitis


\\\\ A 12-year-old boy has left body weakness . An MRI scan reveals a polycystic lesion
The parasitic brain lesion most likely to have a large cyst containing numerous daughter
cysts is that associated with -
\\\ Taenia solium
\\\ Schistosoma haematobium
\\ Taenia echinococcus
\\\ Diphyllobothrim latum
\\\ A 17-year-old right-handed boy has had infectious meningitis 8 times over the past 3
years He has otherwise been healthy and developed normally. Reccurent meningitis
developes in persons with -
\\\ Otitis media
\\\ Epilepsy
\\\ Multiple sclerosisi
\\ Cerebrospinal fluid (CSF) leaks

\\\\ A 72 –year-old right-handed woman has 2 days of headache and fever, followed
worsening confusion. she is taken to the hospital after having a generalized seizure/
head CT is consistent with left temporal hemorrhage and swelling/ Localization of
encephaliis to the medial temporal or orbital frontal regions of the brain is most
consistent with -
\\\ Trponema pallidum
\\\ Varicella zoster virus
\\ Herpes simplex virus
\\\ Cryptococcus neoformans

\\\\ A 72 –year-old right-handed woman has 2 days of headache and fever, followed
worsening confusion. she is taken to the hospital after having a generalized seizure/
head CT is consistent with left temporal hemorrhage and swelling. Neuroimaging of the
brain before attempting a lumbar puncture is advisable in cases of acute encephalitis
because -
\\\ The diagnosis may be evident on the basis of MRI alone
\\ Massive edema in the temporal lobe may make herniation imminent
\\\ The CT picture may determine whether a brain biopsy should be obtained
\\\ Shunting of the ventricules is usually indicated and the imaging studies are needed to
direct the placement of the shunt

\\\\ A 72 –year-old right-handed woman has 2 days of headache and fever, followed
worsening confusion. she is taken to the hospital after having a generalized seizure/
head CT is consistent with left temporal hemorrhage and swelling CSF testing
establishes this case as being the commonest form of acute encephalitis.The CSF
changes late in the course of the disease typically include –
\\ An increased number of lymphocytes
\\\ Glucose content of less than two-thirds the serum level
\\\ A protain content of less than 45 mg/dL
\\\ A normal opening pressure
\\\ A predominance of polymorphonuclear white cells

\\\\ A 72 –year-old right-handed woman has 2 days of headache and fever, followed
worsening confusion. she is taken to the hospital after having a generalized seizure/
head CT is consistent with left temporal hemorrhage and swelling. CSF testing
establishes this case as being the commonest form of acute encephalitis. With this
disease EEG may exhibit -
\\\ Activity over the frontal regions
\\\ Activity over the frontal regions
\\\ Three-per-second spike and wave discharges
\\ Bilateral, periodic epileptiform discharges

\\\\ A 55-year-old woman has a progressive dementia over the past year. Over the last 3
months she has developed dysarthria, myoclonus intention tremor and hyperreflexia.
Her CSF VDRL is positive. This patient’s symptoms are being caused by which of the
following? –
\\\ A response to penicillin treatment
\\\ An autoummune reaction
\\\ An acute meningoencephalitis
\\ A chronic meningoencephalitis

\\\\ A 55-year-old woman has a progressive dementia over the past year. Over the last 3
months she has developed dysarthria, myoclonus intention tremor and hyperreflexia.
Her CSF VDRL is positive This patient’s symptoms are being caused by a chronic
meningoencephalitis. This patients underlying disease may present a picture easily
confused with brain tumor if
\\\ A reaction to penicillin occurs
\\ An intracranial gumma forms
\\\ Tabes dorsales is the primary manifestation of the disease
\\\ Meningovascular syphilis develops

\\\\ A 18-year-old man notices tingling about his ankles 2 weeks after an upper
respiratory tract infection. Within 2 days he has weakness in dorsiflexion of both feet
and within 1 week he develops problems walking. he has no loss of bladder or bowel
control. His weakness progreses rapidly over the ensuing week and necessitates his
being placed on a ventilator to support his breathing. He is quadriplegic but retains
control of his eye movements. cerebrospinal fluid reveal a protain content of greater
than 1g/dL with a normal white cell count. There are no red blood cells in the CSF-
\\\ Subacute sclerosing panencephalitis (SSPE)
\\\ Progressive multifocal leukoencephalopathy
\\\ Rabies encephalitis
\\ Guillain-Barre syndrome

\\\\ A 28-year oldmedical student comes into emergency department complaining of


fever , headache, photophobia and myalgias over the past 12 hours.He has been
studying the neuroprotective effects in hamster model of stroke.Prior to onset of the
current symptoms he had a mild fever that subsided spontaneously and was attributed
to”flu’.Neurological examination reveals nuchal rigidity but is otherwise normall. He has
a mild erythematose rush and mild generelised lymphadenopathy. CSF examination
shows an opening pressure of 280mm h20, while blood cell count 1000/mm(98%
mononuclear), a glucose concentration of 30mg/dL(CSF plazma glucose ratio of 0,3)
and a protein concentration of 60mg/dL. Laboratory tests show mild leukopenia and
thrombocytopenia . Human immunodeficiency virus type 1 (HIV-1) serology is negative.
Which of the following is the most likely etiological agent?-
\\\\ Varicella-zoster virus
\\\ Herpes simplex virus 1(HSV-1)
\\\ West Nile virus
\\ Lymphocytic choriomeningitis virus

\\\\ A 65-years old previously healthyman presents in the emergency room because
ofheadache fever and neck stiffness over the past 12 hours. Examination reveals
temperature of 40oC, pain on flexion of the neck and an apical systolic murmur. a chest
x-ray reveals a left lower lobe pneumonia. Which of the following is the most appropriate
statement regarding dexamethasone treatment in this patient?-
\\\ Dexamethasone is contraindicated, given the presence of presumed bacterial
meningitis, endocarditis and pneumonia
\\\ Dexamethasone shoul be administered if the patient’s mental status deteriorates
despite approriate empiric antimicrobial therapy
\\\ A 10-mg dose of dexamethasone should be given prophylactically against brain
edema after 24 hours of antimicrobial therapy if blood cultures are negative
\\ A 10-mg intravenous (IV) dose of dexamethasone should be given prior to initiation of
antimicrobialtherapy and continued every 6 hours for 4 days

\\\\ A 35-year-old previously healthy men presents to the emergency room complaining
of headache and neck pain over the past 6 hours.Examination reveals temperature of
39,4oC, mild tachycardia, and normal blood pressure. He is mildly somnolent,has pain
in flexion of the neck, but no focal neurological deficits, and has an erythematous
maculopapular rash on the chest and forearms. Which of the following antimicrobial
agents should be added to the empirical regimen with cefepine and vancomycin until
the CSF results are available -
\\\ Ampicillin
\\\ Gentamicin
\\\ Rifampin
\\ Doxycycline

\\\\ A 22-years old college student is brought to the emergency department for
headache, photophobia, agitation, and fever that develops over the course of 6 hours.
Examination reveals hyperthermia of 40oC heart rate of 120 beats per minute and blood
pressure of 80/50 mm Hg. He had petechie in the trunk, conjunctiva and palate. He
received dexamethazone 10 mg intravenousely followed by ceftriaxon and vancomycyn.
CSF examination reveals an opening pressure of 360.. H2O, while blood cells
5000/mm(80%neutrophils), a glucose concentration of 10mg/dL, and a protein
concentrtion of 120 mg/dL. Gram’s stain demonstrates gramnegative diplococci. latx
particle agglutination test detected Neisseria menengitidis antigen. Culture confirms that
the strain is sensitive to penicillin.Which of the following is the recommended specific
antibiotic treatment in this patient? -
\\ Ampicillin intravenously for 5 to 7 days followed by oral rifampin for 2 days
\\\ Nafcillin intravenously for 10 to 21 days followed by oral ampicillin for 3 days
\\\ Ampicillin intravenously for 2 to 3 weeks followed by oral azithromycin for 5 days
\\\ Nafcillin intravenously for 5 to 7 days followed by oral rifampin for 2 days

\\\\ In a patient who is three weeks after c liver transplantation , from listed below
virusess which infection of CNS is most common?-
\\\ Rabies virus
\\\ Human herpes virus 6 (HHV6)
\\\ Varicella-zoster virus
\\ Cytomegalovirus

\\\\ A 68-year old patient with a history of normal pressure hydrocephalus underwent
ventriculoperitoneal shunt 6 months ago, with improvement of gait and cognitive
functions. Over past 2 days he has developed headache, progressive deterioration of
his gait, and increased somnolence. The patient has no historyof diabetes or alcohol
use. Ct scan of the head reveals minimal ventricular enlargement compared with the
scan performed 2 months prior to onset of the symptoms. CSF examination reveals an
opening pressure of360-mm H2O, white blood cells 1000/mm, (80% neutrophiles),a
glucose concentration of 20 mg/dl and a protain concentration of 120 mg/dL Which of
the following is the most likely cause of the symptoms? -
\\\ Streptococcus pneumoniae
\\\ Listeria monocytogenes
\\ Staphylococcus aureus
\\\ Pseudomonas aeruginosa

\\\\ Localization of an encephalitis to the medial temporal or orbital frontal regions of the
brain is the most consistent with-
\\\ Treponema Pallidum
\\\ Varicella zoster virus
\\ Herpes simplex virus
\\\ Cryptococcus neoformans

\\\\ A 22-year-old woman reports a scotoma progressing across her left visual field over
the course of 30 min followed by left hemicranial throbbing pain,nausea and
photophobia. Her brother and mother have similar headaches. Which of the following is
present in classic migraine but not in common migraine? -
\\\ Photophobia
\\\ Familial pattern
\\ Visual aura
\\\ Hemicranial pain

\\\\ Basilar migraine differs from classic migraine in the -


\\\ Sex of the person most often affected
\\\ Resistance of the visual system to involvement
\\ Severity of symptoms
\\\ Duration of aura

\\\\ A 23-year-old woman has had 1 week of worsening facial pain. She describes it as
an intense shooting pain that comes and goes. It is only present on her right face.
Which of the following is most likely to be this patient’s underlying problem?-
\\ Multiple sclerosis
\\\ Tolosa-Hunt syndrome
\\\ Migraine
\\\ Anterior communicating artery aneurism

\\\\ Both trigeminal neuralgia and atypical facial pain involve pain that may be -
\\\ Lancinating
\\\ Paroxysmal
\\\ Associated with anesthetic patches
\\ Unilateral

\\\\ 103.An obese 37-year-old woman complains of daily headache, worse in the
morning, for one year. She has episodes of transient visual obscurations affecting each
eye, and also complains of pulsatile tinnitus. Examination is notable for bilateral
papilledema. There are no other abnormalities.Select the most likely diagnosis -
\\\ Carotid artery dissection
\\ Pseudotumor cerebri
\\\ Glioblastoma multiforme
\\\ Thunderclap headache

\\\\ A 42-year-old man presents with a sudden and severe headache associated with
nausea. The headache reaches maximal intensity within 5s. He has no prior history of
headache. Examination is unremarkable. Computed tomograpfy and spinal fluid
examination show no evidence of blood. He later admits that he had been engaged in
sexual activity when the headache occurred Select the most likely diagnosis-
\\\ Carotid artery dissection
\\\ Pseudotumor cerebri
\\\ Glioblastoma multiforme
\\ Thunderclap headache
\\\Analgesic rebound headach
\\\\ A 29-year-old man relates that he has had recent headaches only when standing up.
The headaches resolve quickly when he lies down, and are accompanied by mild
nausea. His examination is normal Select the most likely diagnosis -
\\\ Pseudotumor cerebri
\\\ Thunderclap headache
\\\ Analgesic rebound headach
\\ Intracranial hypotension

\\\\ Which of the following is typically the most difficult type of pain to treat? -
\\ Psychogenic pain
\\\ Intermittent somatic pain
\\\ Continuous visceral pain
\\\ Intermittent neuropathic pain

\\\\ Which of the following types of headaches is most likely to be responsive to


inhalation of 100% oxygen?-
\\\ Migraine headache
\\\ Tension headache
\\ Cluster headache
\\\ Cervicogenic headache

\\\\ Which of the following statements regarding the pain of multiple sclerosis(MS) is
true?
\\ Trigeminal neuralgia is the most common pain disorder associated with MS
\\\ Most patients with MS have headaches associated with the desease process
\\\ Primary psychogenic pain is common in MS
\\\ The majority of MS patients with pain have central pain associated with the disease
process

\\\\ Headache types more common in women include


\\ tension headache
\\\ giant cell arteriitis
\\\ migrain headache
\\\ cluster headache

\\\\ A 30- year- old man presents with a headache that started yesterday. As he was
shoveling snow yesterday, he felt a sudden pain in the front of his head. The pain does
not throb and has been relatively constant since. He says that now his neck also
become a little stiff. He carries a diagnosis of migraine headaches, but says that this is
different than his usual headaches. He is afebrile and has a normal exam except for
slight photophobia and mild discomfort with neck flexion. Which action should be done
next? -
\\\ Obtain a brain MRI
\\ Obtain a brain CT
\\\ Obtain a cerebral angiogram
\\\ Obtain an EEG

\\\\ A 30- year- old man presents with a headache that started yesterday. As he was
shoveling snow yesterday, he felt a sudden pain in the front of his head. The pain does
not throb and has been relatively constant since. He says that now his neck also
become a little stiff. He carries a diagnosis of migraine headaches, but says that this is
different than his usual headaches. He is afebrile and has a normal exam except for
slight photophobia and mild discomfort with neck flexion. A head scan is normal. What
should be done next? -
\\\ Begin intravenous heparin
\\ Perform a lumber puncture
\\\ Obtain a brain MRI
\\\ Obtain a cerebral angiogram

\\\\ A 30- year- old man presents with a headache that started yesterday. As he was
shoveling snow yesterday, he felt a sudden pain in the front of his head. The pain does
not throb and has been relatively constant since. He says that now his neck also
become a little stiff. He carries a diagnosis of migraine headaches, but says that this is
different than his usual headaches. He is afebrile and has a normal exam except for
slight photophobia and mild discomfort with neck flexion. A head scan is normal. A
lumbar puncture shows 7000 red blood cells in tube 1 and 7200 in tube 4. There are 9
white blood cells in each. The fluid is xanthochromic. The next step in managing this
case to
\\ Arrange for a cerebral angiogram and call a neurosurgical consult
\\\ Give a patient a prescription for sumatriptan and send him home
\\\ Immediately give 2g of intravenous ceftriaxone
\\\ Immediately start intravenous acyclovir

\\\\ The most definitive test for identifying intracranial aneurisms is -


\\\ MRI scanning
\\\ CT
\\\ PET
\\ Cerebral angiography

\\\\ A 21 year old woman presents with right arm loss of sensation that has been
progressive over a few days. Her physician is concerned that this might be some type of
demyelinating disorder. A relatively small plaque of demyelination should be evident on
which of the following -
\\\ T1-weighted MRI
\\ T2-weighted MRI
\\\ Precontrast CT
\\\ Diffusion-weighted MRI

\\\\ A 23-year –old woman complains of 2 days of visual loss associated with discomfort
in the right eye. She appears otherwise healthy, but her family reports recurrent
problems with bladder control over the prior 2 years, which she is reluctant to discuss.
On neurologic examination this young women exhibits dysmetria in her right arm, a
plantar extensor response of the left foot and slurred speech. The most informative
ancillary test would be
\\\ Visual evoked response testing
\\\ Sural nerve biopsy
\\\ EEG
\\ MRI

\\\\ A 16 year-old boy is struck on the side of the head by a bottle thrown by a friend. He
appears dazed for about 30 s, but is apparently lucid for several minutes before he
abruptly becomes stuporous. His limbs on the side opposite the site of the blow are
more flaccid than those on the same side as the injury. On arrival in the emergency
room 25 min after the accident, he is unresponsive to painfull stimuli. His pulse is
40/min, with an ECG revealing no arrythmias. His blood pressure in both hands is
170/100 mm Hg. Although papilledema is not evident in his fundi, he has venous
distention and absent pulsation of the retinal vasculature. The best explanation for this
young man’s evolving clinical signs is
\\\ A seizure disorder
\\\ A cardiac conduction defect
\\ Increased intracranial pressure
\\\ Sick sinus syndrome

\\\\ A 16 year-old boy is struck on the side of the head by a bottle thrown by a friend. He
appears dazed for about 30 s, but is apparently lucid for several minutes before he
abruptly becomes stuporous. His limbs on the side opposite the site of the blow are
more flaccid than those on the same side as the injury. On arrival in the emergency
room 25 min after the accident, he is unresponsive to painfull stimuli. His pulse is
40/min, with an ECG revealing no arrythmias. His blood pressure in both hands is
170/100 mm Hg. Although papilledema is not evident in his fundi, he has venous
distention and absent pulsation of the retinal vasculature. The best explanation for this
young man’s evolving clinical signs is increased intracranial pressure. The wisest
management over the next 4 h for the patient is -
\\ Craniotomy
\\\ Antihypertensive medication
\\\ Transvenous pacemaker placement
\\\ Ventriculoperitoneal shunt

\\\\ A 16 year-old boy is struck on the side of the head by a bottle thrown by a friend. He
appears dazed for about 30 s, but is apparently lucid for several minutes before he
abruptly becomes stuporous. His limbs on the side opposite the site of the blow are
more flaccid than those on the same side as the injury. On arrival in the emergency
room 25 min after the accident, he is unresponsive to painfull stimuli. His pulse is
40/min, with an ECG revealing no arrythmias. His blood pressure in both hands is
170/100 mm Hg. Although papilledema is not evident in his fundi, he has venous
distention and absent pulsation of the retinal vasculature. MRI the patient’s head within
the first few hours of injury should reveal -
\\\ A normal brain
\\\ Intracerebral hematoma
\\\ Temporallobe contusion
\\ Epidural hematoma

\\\\ A 16 year-old boy is struck on the side of the head by a bottle thrown by a friend. He
appears dazed for about 30 s, but is apparently lucid for several minutes before he
abruptly becomes stuporous. His limbs on the side opposite the site of the blow are
more flaccid than those on the same side as the injury. On arrival in the emergency
room 25 min after the accident, he is unresponsive to painfull stimuli. His pulse is
40/min, with an ECG revealing no arrythmias. His blood pressure in both hands is
170/100 mm Hg. Although papilledema is not evident in his fundi, he has venous
distention and absent pulsation of the retinal vasculature. The best explanation for this
young man’s evolving clinical signs is increased intracranial pressure. CT of the patients
head within 2 h of the injury should reveal -
\\\ A normal brain
\\ A lens-shaped density over the frontal lobe
\\\ Increased CSF density with a fluid-fluid level
\\\ Multifocal attenuation of cortical tissue

\\\\ The elderly person who suffers relatively mild head trauma but subsequently
develops a progressive dementia over the course of several weeks is most likely to
have sustained which of the following
\\\ An acute subdural hematoma
\\\ An acute epidural hematoma
\\ A chronic subdural hematoma
\\\ An intracerebral hematoma

\\\\ The most common sources of primary brain tumors are -


\\ Glial cells
\\\ Neurons
\\\ Meningeal cells
\\\ Lymphocytes

\\\\ The most common type of primary brain tumors are -


\\\ Meningioma
\\ Astrocytoma
\\\ Lymphosarcoma
\\\ Oligodendroglioma

\\\\ A 72-year-old woman has a head CT performed because of headaches. It is


significant for a left hemisphere mass with an overlying hyperostosis of the skull. She
most likely has which of the following -
\\ Meningioma
\\\ Pituitary adenoma
\\\ Astrocytoma
\\\ Schwannoma

\\\\ A37-year-old man presents with visual impairment. Examination reveals a bitemporal
hemianopsia. Which of the following tumors is most likely responsible for finding -
\\\ Optic glioma
\\\ Occipital astrocytoma
\\\ Brainstem glioma
\\ Pituitary adenoma

\\\\ With an ependimoma of the posterior fossa, the patient is at risk of dying because of -
\\\ Embolifrom the tumor
\\\ Vascular occlusion by the tumor
\\\ Hemorrhagic necrosis of the tumor

\\\\ A pure motor stroke is most likely with damage to the


\\ Internal capsule
\\\ Cerebellum
\\\ Putamen
\\\ Caudate

\\\\ A pure sensory stroke is most likely with damage to the


\\\ internal capsule
\\ Thalamus
\\\ Hippocampus
\\\ Globus pallidus

\\\\ Occlusion of which of the following arteries typically produces lateral


medullary infarction syndrome?
\\\ Basilar artery
\\ Vertebral artery
\\\ Superior cerebellar artery
\\\ Anterior inferior cerebellar artery(AICA)
\\\\ A 72-year-old woman has the abrupt onset of the right face and hand weakness,
disturbed speech production and a right homonymous hemianopsia. This is most likely
attributable to occlusion of the
\\ Left middle cerebral artery
\\\ Left anterior cerebral artery
\\\ Left vertebrobasilar artery
\\\ Right anterior choroidal artery

\\\\ A39-year-old woman has diplopia several times a day for 6 weeks She consults a
physician when the double vision becomes unremitting, and also complains of dull pain
behind her right eye. When a red glass is placed over her right eye and she is asked to
look at a flashlight off to her left, she reports seeing a white light and a red light. The red
light appears to her to be more to the left than the white light . Her right pupil is more
dilated than her left pupil and responds less briskly to a bright light directed at it than
does the left pupil. Before any further investigations can be performed, the woman
develops the worst headache in her life and becomes stuporous. Her physician
discovers that she has marked neck stiffness and photophobia. The physician performs
a transfemoral angiogram. The radiologic study is expected to reveal that the woman
has
\\\ An arterionenous malformation
\\\ An occipital astrocytoma
\\\ A sphenoidal meningeoma
\\ A saccular aneurism

\\\\ A 39-year-old woman has diplopia several times a day for 6 weeks She consults a
physician when the double vision becomes unremitting, and also complains of dull pain
behind her right eye. When a red glass is placed over her right eye and she is asked to
look at a flashlight off to her left, she reports seeing a white light and a red light. The red
light appears to her to be more to the left than the white light . Her right pupil is more
dilated than her left pupil and responds less briskly to a bright light directed at it than
does the left pupil. The cranial nerve injury likely to be responsible for all these
observations is one involving
\\\ The second cranial nerve
\\ The third cranial nerve
\\\ The fourth cranial nerve
\\\ The sixth cranial nerve

\\\\ A 39-year-old woman has diplopia several times a day for 6 weeks She consults a
physician when the double vision becomes unremitting, and also complains of dull pain
behind her right eye. When a red glass is placed over her right eye and she is asked to
look at a flashlight off to her left, she reports seeing a white light and a red light. The red
light appears to her to be more to the left than the white light . Her right pupil is more
dilated than her left pupil and responds less briskly to a bright light directed at it than
does the left pupil The site of the lesion responsible for this woman’s symptoms and
signs is most probably the
\\\ Anterior communicating artery
\\ Posterior communicating artery
\\\ Anterior cerebral artery
\\\ Middle cerebral artery

\\\\ A 39-year-old woman has diplopia several times a day for 6 weeks She consults a
physician when the double vision becomes unremitting, and also complains of dull pain
behind her right eye. When a red glass is placed over her right eye and she is asked to
look at a flashlight off to her left, she reports seeing a white light and a red light. The red
light appears to her to be more to the left than the white light . Her right pupil is more
dilated than her left pupil and responds less briskly to a bright light directed at it than
does the left pupil. Three days after developing stiffness and photophobia, the woman
develops left-sided weakness and hyperreflexia. her left plantar response is upgoing.
Her physician presumes that these deficits are a delayed effect of the subarachnoid
blood and so would treat her with
\\\ Heparin
\\\ Warfarim
\\ Nimodipine
\\\ Phenytoin

\\\\ A 73-year –old man with a history of hypertension complains of a 10-min episodes of
left-sided weakness and slurred speech. On further questioning he relates three brief
episodes in the last month of sudden impairment of vision affecting the right eye. His
examination now is normal.
Which of the following would be the most appropriate next diagnostic test?
\\\ Creatine phosphokinase
\\\ Holter monitor
\\\ Visual evoked potentials
\\ Carotid artery Doppler ultrasound

\\\\ A 73-year –old man with a history of hypertension complains of a 10-min episodes of
left-sided weakness and slurred speech. On further questioning he relates three brief
episodes in the last month of sudden impairment of vision affecting the right eye. His
examination now is normal. The episodes of visual loss are most likely related to
\\\ Retinal vein thrombosis
\\ Central retinal artery ischemia
\\\ Posterior cerebral artery ischemia
\\\ Middle cerebral artery ischemia

\\\\ A 73-year –old man with a history of hypertension complains of a 10-min episodes of
left-sided weakness and slurred speech. On further questioning he relates three brief
episodes in the last month of sudden impairment of vision affecting the right eye. His
examination now is normal. A thorough evaluation reveals that patient has a 90%
stenosis of the right internal carotid artery at the bifurcation. The management option
most likely to prevent a future stroke is which of the following?
\\\ Warfarin
\\\ Carotid artery angioplasty
\\ Carotid endarterectomy
\\\ Extracranial-intracranial bypass

\\\\ A 62-year old man with a history of myocardial infarction awakens with a dense right
sided hemiplegia. His eyes are tonically deviated to the left and he does not respond to
threat on the right side of his visual field. He appears to be alert and responds to pain
on the left side of his body. His speech is unintelligible and nonfluent and he follows no
instructions. Efforts to get him to repeat simple phrases consistently fail. Pick the
language disturbance that best explains the clinical picture
\\\ Broca’s aphasia
\\\ Wernicke’s aphasia
\\\ Anomic aphasia
\\ Global aphasia

\\\\ A 45-year-old woman with chronic atrial fibrillation discontinues warfarin treatment
and abruptly develops problems with language comprehension. She is able to produce
some intelligible phrases and produces sound quite fluently; however, she is unable to
follow simple instructions or to repeat simple phrases On attempting to write, she
becomes very frustrated and agitated. Emergency MRI reveals a lesion of the left
temporal lobe, that extends into the superior temporal gyrus Pick the language
disturbance that best explains the clinical picture
\\\ Broca’s aphasia
\\ Wernicke’s aphasia
\\\ Anomic aphasia
\\\ Global aphasia

\\\\ A 71-year-old man develops headache and slight difficulty speaking while having
sexual intercourse. He has a long-standing history of hypertension, but has been on
medication for more than 7 years. He makes frequent errors in finding words and
follows complex commands somewhat inconsistently. The most obvious defect in his
language function is his inability to repeat the simplest of phrases without making
repeated errors. An emergency CT scan reveals an intracerebral hemorrhage in the left
parietal lobe that appears to communicate with the lateral ventricle Pick the language
disturbance that best explains the clinical picture
\\\ Broca’s aphasia
\\\ Wernicke’s aphasia
\\ Conduction aphasia
\\\ Mixed transcortical apasia

\\\\ A 24-year-old woman abruptly loses all speech during the third trimester of otherwise
uncomplicated pregnancy. She has a history of severe migraines during which she
occasionally develops a transient right hemiplegia. Her comprehension is good, and she
is frustrated by her inability to speak or write. She is unable to repeat simple phrases,
but she does begin to produce simple words within 5 days of the acute disturbance of
language Pick the language disturbance that best explains the clinical picture
\\ Broca’s aphasia
\\\ Wernicke’s aphasia
\\\ Global aphasia
\\\ Conduction aphasia

\\\\ A 75-year old man with a history of recent memory impairment is admitted with
headache, confusion, and a left homonymous hemianopsia. He has recently had two
episodes of brief unresponsiveness. There is no history of hypertension. Computed
tomography (CT) scan shows a right occipital lobe hemorrhage with some subarachnoid
extension of the blood. An MRI scan with gradient echo sequence reveals foci of
hemosiderin in the right temporal and left frontal cortex. The likely cause of this patient’s
symptoms and signs is
\\\ Gliomatosis cerebri
\\\ Multi-infarct-dementia
\\ Amyloid angiopathy
\\\ Undiagnosed hypertension

\\\\ A 61-year-old man with a history of hypertension has been in excellent health until
he presents with vertigo and , unsteadiness lasting for two days. He then develops
nausea, vomiting, dysphagia, hoarseness, ataxia, left facial pain, and right-sided
sensory loss. There is no weakness. On examination, he is alert, with a normal mental
status. He vomits with head movement. There is skew deviation of the eyes, left ptosis,
clumsiness of the left arm, and titubation. He has loss of pain and temperature
sensation in the left foot. He is unable to walk. Magnetic resonance imaging (MRI) in
this patient might be expected to show which of the following?
\\\ Basilar artery tip aneurysm
\\\ Right lateral medullary infarction
\\ Left lateral medullary infarction
\\\ Left medial medullary infarction

\\\\ A 61-year-old man with a history of hypertension has been in exellent health until he
presents with vertigo and , unsteadiness lasting for two days. He then develops nausea,
vomiting, dysphagia, hoarseness, ataxia, left facial pain, and right-sided sensory loss.
There is no weakness. On examination, he is alert, with a normal mental status. He
vomits with head movement. There is skew deviation of the eyes, left ptosis, clumsiness
of the left arm, and titubation. He has loss of pain and temperature sensation in the left
foot. He is unable to walk. Right sided sensory loss is secondary to involvement of
which of the following structures?
\\\ The right lateral spinothalamic tract
\\ The left.lateral spinothalamic tract
\\\ Ninth and Tenth cranial nerves
\\\ Inferior cerebellar peduncle

\\\\ The most striking neurologic complication of von Economos encephalitis


(encephalitis lethargica), a type of encephalitis that occurred in epidemic proportions
along with viral influenca between 1917 and 1928 was
\\\ Blindness
\\\ Hearing loss
\\\ Paraplegia
\\ Parkinsonism

\\\\ A17-year-old right-handed boy has had infectious meningitis 8 times over the past 3
years He has otherwise been healthy and developed normally. Reccurent meningitis
develops in persons with
\\\ Ottitis media
\\\ Epilepsy
\\\ Multiple sclerosisi
\\ Cerebrospinal fluid (CSF) leaks

\\\\ A 72 –year-old right-handed woman has 2 days of headache and fever, followed by
worsening confusion. she is taken to the hospital after having a generalized seizure/ A
head CT is consistent with left temporal hemorrhage and swelling/ Localization of
encephalitis to the medial temporal or orbital frontal regions of the brain is most
consistent with
\\\ Trponema pallidum
\\\ Varicella zoster virus
\\ Herpes simplex virus
\\\ Cryptococcus neoformans

\\\\ A 72 –year-old right-handed woman has 2 days of headache and fever, followed by
worsening confusion. she is taken to the hospital after having a generalized seizure/ A
head CT is consistent with left temporal hemorrhage and swelling. Neuroimaging of the
brain before attempting a lumbar puncture is advisable in cases of acute encephalitis
because
\\\ The diagnosis may be evident on the basis of MRI alone
\\ Massive edema in the temporal lobe may make herniation imminent
\\\ The CT picture may determine whether a brain biopsy should be obtained
\\\ Shunting of the ventricules is usually indicated and the imaging studies are needed to
direct the placement of the shunt
\\\\ A 72 –year-old right-handed woman has 2 days of headache and fever, followed by
worsening confusion. she is taken to the hospital after having a generalized seizure/ A
head CT is consistent with left temporal hemorrhage and swelling CSF testing
establishes this case as being the commonest form of acute encephalitis. The CSF
changes late in the course of the disease typically include
\\ An increased number of lymphocytes
\\\ A glucose content of less than two-thirds the serum level
\\\ A protain content of less than 45 mg/dL
\\\ A normal opening pressure

\\\\ A 72 –year-old right-handed woman has 2 days of headache and fever, followed by
worsening confusion. she is taken to the hospital after having a generalized seizure/ A
head CT is consistent with left temporal hemorrhage and swelling. CSF testing
establishes this case as being the commonest form of acute encephalitis. With this
disease EEG may exhibit
\\\ alfa - activity over the frontal regions
\\\ beta- activity over the frontal regions
\\\ Three-per-second spike and wave discharges
\\ Bilateral, periodic epileptiform discharges

\\\\ A 55-year-old woman has a progressive dementia over the past year. Over the last 3
months she has developed dysarthria, myoclonus intention tremor and hyperreflexia.
Her CSF VDRL is positive This patient’s symptoms are being caused by a chronic
meningoencephalitis. This patients underlying disease may present a picture easily
confused with brain tumor if
\\\ A reaction to penicillin occurs
\\ An intracranial gumma forms
\\\ Tabes dorsalis is the primary manifestation of the disease
\\\ Meningovascular syphilis develops

\\\\ A 18-year-old man notices tingling about his ankles 2 weeks after an upper
respiratory tract infection. Within 2 days he has weakness in dorsiflexion of both feet
and within 1 week he develops problems walking. he has no loss of bladder or bowel
control. His weakness progress rapidly over the ensuing week and necessitates his
being placed on a ventilator to support his breathing. He is quadriplegic but retains
control of his eye movements. cerebrospinal fluid reveal a protein content of greater
than 1g/dL with a normal white cell count. There are no red blood cells in he CSF
\\\ Subacute sclerosing panencephalitis(SSPE)
\\\ Progressive multifocal leukoencephalopathy
\\\ Rabies encephalitis
\\ Guillain-Barre syndrome

\\\\ A 65-years old previously healthy man presents in the emergency room because
ofheadache fever and neck stiffness over the past 12 hours. Examination reveals
temperature of 40o C, pain on flexion of the neck and an apical systolic murmur. a chest
x-ray reveals a left lower lobe pneumonia. Which of the following is the most appropriate
statement regarding dexamethasone treatment in this patient?
\\\ Dexamethasone is contraindicated, given the presence of presumed bacterial
meningitis, endocarditis and pneumonia
\\\ Dexamethasone should be administered if the patient’s mental status deteriorates
despite appropriate empiric antimicrobial therapy
\\\ A 10-mg dose of dexamethasone should be given prophylactically against brain
edema after 24 hours of antimicrobial therapy if blood cultures are negative
\\ A 10-mg intravenous (IV) dose of dexamethasone should be given prior to initiation of
antimicrobial therapy and continued every 6 hours for 4 days

\\\\ A 22-years old college student is brought to the emergency department for
headache, photophobia, agitation, and fever that develops over the course of 6 hours.
Examination reveals hyperthermia of 40 o C heart rate of 120 beats per minute and
blood pressure of 80/50 mm Hg. He had petechie in the trunk, conjunctiva and palate.
He received dexamethazone 10 mg intravenously followed by ceftriaxon and
vancomycyn. CSF examination reveals an opening pressure of 360.. H2O, while blood
cells 5000/mm(80% neutrophils), a glucose concentration of 10mg/dL, and a protein
concentrtion of 120 mg/dL. Gram’s stain demonstrates gram-negative diplococci. Latex
particle agglutination test detected Neisseria menengitidis antigen. Culture confirms that
the strain is sensitive to penicillin. Which of the following is the recommended specific
antibiotic treatment in this patient?
\\ Ampicillin intravenously for 5 to 7 days followed by oral rifampin for 2 days
\\\ Nafcillin intravenously for 10 to 21 days followed by oral ampicillin for 3 days
\\\ Ampicillin intravenously for 2 to 3 weeks followed by oral azithromycin for 5 days
\\\ Cefotaxime intravenously for 3 days followed by oral azythromycin for 5 days

\\\\ In a patient who is three weeks from orthotopic liver transplantation , from which of
the following viruses is CNS infection most common?
\\\ Rabies virus
\\\ Human herpervirus 6(HHV6)
\\\ Varicella-zoster virus
\\ Cytomegalovirus

\\\\ A 68-year old patient with a history of normal pressure hydrocephalus underwent a
ventriculoperitoneal shunt 6 months ago, with improvement of gait and cognitive
functions. Over past 2 days he has developed headache, progressive deterioration of
his gait, and increased somnolence. The patient has no history of diabetes or alcohol
use. CT scan of the head reveals minimal ventricular enlargement compared with the
scan performed 2 months prior to onset of the symptoms. CSF examination reveals an
opening pressure of360-mm H2O, white blood cells 1000/mm,(80%neutrophiles),a
glucose concentration of 20 mg/dl and a protein concentration of 120 mg/dL Which of
the following is the most likely cause of the symptoms?
\\\ Streptococcus pneumoniae
\\\ Listeria monocytogenes
\\ Staphylococcus aureus
\\\ Escherichia coli

\\\\ Localization of an encephalitis to the medial temporal or orbital frontal regions of the
brain is the most consistent with
\\\ Treponema Pallidum
\\\ Varicella zoster virus
\\ Herpes simplex virus
\\\ Cryptococcus neoformans

\\\\ A 22-year-old woman reports a scotoma progressing across her left visual field over
the course of 30 min followed by left hemicranial throbbing pain, nausea and
photophobia. Her brother and mother have similar headaches. Which of the following is
present in classic migraine but not in common migraine?
\\\ Photophobia
\\\ familial pattern
\\ Visual aura
\\\ Hemicranial pain

\\\\ Basilar migraine differs from classic migraine in the


\\\ Sex of the person most often affected
\\\ Resistance of the visual system to involvement
\\ Severity of symptoms
\\\ Duration of aura

\\\\ Both trigeminal neuralgia and atypical facial pain involve pain that may be
\\\ Lancinating
\\\ Paroxysmal
\\\ Associated with anesthetic patches
\\ Unilateral

\\\\ An obese 37-year-old woman complains of daily headache, worse in the morning, for
one year. She has episodes of transient visual obscurations affecting each eye, and
also complains of pulsatile tinnitus. Examination is notable for bilateral papilledema.
There are no other abnormalities. Select the most likely diagnosis
\\\ Carotid artery dissection
\\ Pseudotumor cerebri
\\\ Glioblastoma multiforme
\\\ Thunderclap headache

\\\\ Which of the following types of headaches is most likely to be responsive to


inhalation of 100% oxygen?
\\\ Migraine headache
\\\ Tension headache
\\ Cluster headache
\\\ Cervicogenic headache

\\\\ A 30- year- old man presents with a headache that started yesterday. As he was
shoveling snow yesterday, he felt a sudden pain in the front of his head. The pain does
not throb and has been relatively constant since. He says that now his neck also
become a little stiff. He carries a diagnosis of migraine headaches, but says that this is
different than his usual headaches. He is afebrile and has a normal exam except for
slight photophobia and mild discomfort with neck flexion. Which action should be done
next?
\\\ Obtain a brain MRI
\\ Obtain a brain CT
\\\ Obtain a cerebral angiogram
\\\ Obtain an EEG

\\\\ A 30- year- old man presents with a headache that started yesterday. As he was
shoveling snow yesterday, he felt a sudden pain in the front of his head. The pain does
not throb and has been relatively constant since. He says that now his neck also
become a little stiff. He carries a diagnosis of migraine headaches, but says that this is
different than his usual headaches. He is afebrile and has a normal exam except for
slight photophobia and mild discomfort with neck flexion. A head scan is normal. What
should be done next?
\\\ Begin intravenous heparin
\\ Perform a lumber puncture
\\\ Obtain a brain MRI
\\\ Obtain a cerebral angiogram
\\\\ The most definitive test for identifying intracranial aneurisms is
\\\ MRI scanning
\\\ CT
\\\ PET
\\ Cerebral angiography

\\\\ A 23-year –old woman complains of 2 days of visual loss associated with discomfort
in the right eye. She appears otherwise healthy, but her family reports recurrent
problems with bladder control over the prior 2 years, which she is reluctant to discuss.
On neurologic examination this young women exhibits dysmetria in her right arm, a
plantar extensor response of the left foot and slurred speech. The most informative
ancillary test would be
\\\ Visual evoked response testing
\\\ Sural nerve biopsy
\\\ EEG
\\ MRI

\\\\ A 16 year-old boy is struck on the side of the head by a bottle thrown by a friend. He
appears dazed for about 30 s, but is apparently lucid for several minutes before he
abruptly becomes stuporous. His limbs on the side opposite the site of the blow are
more flaccid than those on the same side as the injury. On arrival in the emergency
room 25 min after the accident, he is unresponsive to painful stimuli. His pulse is
40/min, with an ECG revealing no arrythmias. His blood pressure in both hands is
170/100 mm Hg. Although papilledema is not evident in his fundi, he has venous
distention and absent pulsation of the retinal vasculature. The best explanation for this
young man’s evolving clinical signs is
\\\ A seizure disorder
\\\ A cardiac conduction defect
\\ Increased intracranial pressure
\\\ Communicating hydrocephalus

\\\\ 16 year-old boy is struck on the side of the head by a bottle thrown by a friend. He
appears dazed for about 30 s, but is apparently lucid for several minutes before he
abruptly becomes stuporous. His limbs on the side opposite the site of the blow are
more flaccid than those on the same side as the injury. On arrival in the emergency
room 25 min after the accident, he is unresponsive to painful stimuli. His pulse is
40/min, with an ECG revealing no arrythmias. His blood pressure in both hands is
170/100 mm Hg. Although papilledema is not evident in his fundi, he has venous
distention and absent pulsation of the retinal vasculature. The best explanation for this
young man’s evolving clinical signs is increased intracranial pressure. The wisest
management over the next 4 h for the patient is
\\ Craniotomy
\\\ Antihypertensive medication
\\\ Transvenous pacemaker placement
\\\ Ventriculoperitoneal shunt

\\\\ A 16 year-old boy is struck on the side of the head by a bottle thrown by a friend. He
appears dazed for about 30 s, but is apparently lucid for several minutes before he
abruptly becomes stuporous. His limbs on the side opposite the site of the blow are
more flaccid than those on the same side as the injury. On arrival in the emergency
room 25 min after the accident, he is unresponsive to painful stimuli. His pulse is
40/min, with an ECG revealing no arrythmias. His blood pressure in both hands is
170/100 mm Hg. Although papilledema is not evident in his fundi, he has venous
distention and absent pulsation of the retinal vasculature. MRI of the patient’s head
within the first few hours of injury should reveal
\\\ A normal brain
\\\ Intracerebral hematoma
\\\ Temporallobe contusion
\\ Epidural hematoma

\\\\ A 16 year-old boy is struck on the side of the head by a bottle thrown by a friend. He
appears dazed for about 30 s, but is apparently lucid for several minutes before he
abruptly becomes stuporous. His limbs on the side opposite the site of the blow are
more flaccid than those on the same side as the injury. On arrival in the emergency
room 25 min after the accident, he is unresponsive to painful stimuli. His pulse is
40/min, with an ECG revealing no arrythmias. His blood pressure in both hands is
170/100 mm Hg. Although papilledema is not evident in his fundi, he has venous
distention and absent pulsation of the retinal vasculature. The best explanation for this
young man’s evolving clinical signs is increased intracranial pressure. CT of the patients
head within 2 h of the injury should reveal
\\\ A normal brain
\\ A lens-shaped density over the frontal lobe
\\\ Increased CSF density with a fluid-fluid level
\\\ Multifocal attenuation of cortical tissue

\\\\ The elderly person who suffers relatively mild head trauma but subsequently
develops a progressive dementia over the course of several weeks is most likely to
have sustained which of the following
\\\ An acute subdural hematoma
\\\ An acute epidural hematoma
\\ A chronic subdural hematoma
\\\ An intracerebral hematoma

\\\\ Which of the following is the mediator of extrapiramidal system


\\\ bradikinine
\\\ epinephrine
\\ acetilcholine
\\\ serotonine

\\\\ Which of the following belongs to Parkinson’s disease symptoms?


\\\ muscle spasticity
\\ hypokinesia
\\\ intention tremor
\\\ tremor ofd a head

\\\\ Which of the following is characteristic for Parkinson’s disease?


\\\ myoclonus
\\ micrographia
\\\ ballism
\\\ athetosis

\\\\ Which of the following is most disabling symptom of Parkinson’s disease?


\\\ tremor
\\\ muscle rygidity
\\ hypokinesia
\\\ shiallorea
\\\\ Which of the following is true in case of Parkinson’s disease?
\\\ usually it is started with tremor
\\ usually asymmetric onset
\\\ postural disturbances from the beginning
\\\ motor fluctuations at the beginning

\\\\ In Parkinson’s disease most prominent neuropathological changes are in


\\\ thalamus
\\ substantia nigra pars compacta
\\\ putamen
\\\ globus pallidus

\\\\ Which of the following is characteristic for Huntington’s disease?


\\\ cerebellar ataxia
\\ choreatic hyperkinesis
\\\ spastic tetraparesis
\\\ myorithmias

\\\\ Which of the following is characteristic for Huntington’s disease?


\\ dementia and chorea
\\\ dementia and spinocerebellar ataxia
\\\ dementia and paraparesis
\\\ dementia and truncal dystonia

\\\\ Juvenile form of Huntington’s disease is characterized by


\\\ severe choreatic hyperkinesis
\\ akinesia and rigidity
\\\ truncal dystonia
\\\ cortical myoclonus

\\\\ Sidenham’s chorea is a disease of


\\\ 2-6 years old children
\\ 6-13 years old children
\\\ 13-17 years old adolescents
\\\ adult people

\\\\ Sidenham’s chorea is associated with


\\\ meningococcal infection
\\\ herpetic encephalitias
\\ rheumatism
\\\ staphylococcal infection

\\\\ Diagnosis of Sidenham’s chorea is confirmed by


\\ high titer of antistreptococcal antibodies in blood and streptococcs in nasopharyngeal smear
\\\ high titer of staphilococcs in blood and nasopharyngeal smear
\\\ CSF pleocytosis
\\\ elevated protein level in CSF

\\\\ What is the correct management scheme for Sidenham’s chorea ?


\\ symptomatic treatment and long-lasting treatment with adequate dose of penicillin
\\\ only several months of symptomatic treatment
\\\ several years of neuroleptic therapy
\\\ several years of antyconvulsive therapy
\\\\ In Wilson’s disease there is disregulatopn of turnover of
\\ Cu
\\\ Fe
\\\ Zn
\\\ Ca

\\\\ Most vulnearable to Cu intoxication part of brain is:


\\\ thalamus
\\\ cranial nerve’s nuclei
\\ basal ganglia
\\\ cerebellar nuclei

\\\\ In Wilson’s disease Cu is primarily accumulated in:


\\\ kidney
\\ hepatic
\\\ splenium
\\\ skin

\\\\ Wilson’s disease may start at the age


\\\ before 10 years
\\\ before 20 years
\\\ at 20-50 years
\\ at any years before 50

\\\\ Enhansed phisiological tremor is seen in


\\\ Parkinson’s disease
\\ thireotoxikosis
\\\ generalised atherosclerosis
\\\ multiple sclerosis

\\\\ Essential tremor is characterised by


\\ postural tremor in extremities
\\\ muscle rigidity
\\\ hypokinesia
\\\ disequilibrium

\\\\ Most frequent cause of hemiballism is


\\\ brain stem encephalitis
\\\ hemorrhage in contralateral internal capsule
\\ stroke with involvement of contralateral subthalamic nucleus
\\\ temporal lobe damage due to herpetic encephalitis

\\\\ The treatment with antiepileptic drugs (AEDs) was carried out with 20 years old
woman with generalized tonic-clonic seizures. The seizures were stopped. She has gait
disorder. This is adverse event of the following AED:
\\\ Primidone
\\\ Valproic acid
\\ Phenytoine
\\\ Carbamazepine

\\\\ The following drug can be used for the treatment of absence epilepsy:
\\\ Phenytoine
\\ Ethosuximide
\\\ Phenobarbitale
\\\ Carbamazepine

\\\\ The following drug is non-effective for treatment of focal epilepsy:


\\\ Vaplroic acid
\\\ Carbamazepine
\\\ Phenobarbitale
\\ Ethosuximide

\\\\ The following drug is non-effective for the treatment of myoclonic seizures:
\\\ Valproic acid
\\ Ethosuximide
\\\ Clonazepame
\\\ All of them

\\\\ The following antiepileptic drug can cause psychosis:


\\\ Phenobarbitale
\\\ Ethosuximide
\\\ Phenytoine
\\ Vigabatrine

\\\\ The teratogenic effect with development of neural tube defect has the following
antiepileptic drug:
\\\ Ethosuximide
\\\ Vigabatrine
\\\ Phenobarbitale
\\ Valproic acid

\\\\ The following antiepileptic drug can be used for intravenous infusion as a drug of first
choice for the treatment of epileptic status:
\\\ Chlorpromazine
\\ Diazepame
\\\ Tranilcipromine
\\\ Ethosuximide

\\\\ The man with 15 year duration of epilepsy referred to urgent department with
generalized tonic-clonic seizures with continous epileptic seizures. The drug of first
choice is:
\\\ Midazolame
\\ Diazepame
\\\ Alprazolame
\\\ Ethosuximide

\\\\ The following is the reason of idiopathic epileps


\\ Genetic predisposition
\\\ Congenital malformation of brain
\\\ The developmental disorder of brain
\\\ Brain trauma

\\\\ The following is the sign of cryptogenic epilepsy:


\\\ Non genetic disorders confirmed by neuroimagi
\\\ The developmental disorder of brain
\\\ Genetic predisposition
\\ Focal epileptic seizures due to brain disorder not confirmed by neuroimaging
\\\\ Average prevalence of epilepsy in the world is:
\\ 1%
\\\ 3%
\\\ 0.5%
\\\ 5%

\\\\ Alpha is EEG activity with frequency:


\\\ 1-4 Hz
\\ 8-13 Hz
\\\ 4-8 Hz
\\\ > 13 Hz

\\\\ Delta is EEG activity with frequency:


\\ 1-4 Hz
\\\ 8-13 Hz
\\\ 4-8 Hz
\\\ > 13 Hz

\\\\ Focal slow wave activity on EEG is the sign of:


\\ Local damage of brain
\\\ Thalamocortical dysfunction
\\\ Diffuse encephalopathy
\\\ Anoxic lesion of brain

\\\\ Intermittent rhythmic EEG activity is the sign of:


\\\ Local damage of brain
\\ Thalamocortical dysfunction
\\\ Diffuse encephalopathy
\\\ Anoxic damage of brain

\\\\ The EEG activity of low amplitude occurs during:


\\\ Local damage of brain
\\\ Thalamocortical dysfunction
\\\ Diffuse encephalopathy
\\ Anoxic damage of brain

You might also like